You are on page 1of 111

Anglo-Chinese Junior College

H2 Mathematics 9740
2008 JC 2 PRELIM PAPER 1 Solutions

1
( )( )
1
1
2
2
2 1
2 3 1
3 1 3
x
x x
x x

| |
= +
|
+ \ .


1
1 1
2
2 2
1 1
2 1
3 3
x x
x

| |
| |
= +
| |
\ .
\ .

1 1
2 2
2
1 1 1
2 1
6 24 3
x x
x x
| |
| |
= +
| |
\ .
\ .

1 1 3
2 2 2
1 13 3
6 8 3
x x x
| |
= +
|
\ .


range of validity:
1
3
x > or
1
3
x <


but since
1
3
x < will result in the sq rt of a negative
number, reject
1
3
x < .
range of validity:
1
3
x > .







2 no. of terms in the sum: 2 n +
sum = ( ) ( )
2 1 1
1
2 2 2
n
m d m n d
+ ( | | | |
+ + + +
| | (
\ . \ .

= ( )
2
1 2 1
2
n
m n d
+
+ + (



3
( )
( )
2 cos 2 1
cos 1 1
2
1
cos 2
2
x x
x x
x
e x dx e dx
e x e dx
+
=
=
} }
}

( )
1
1
1
1
2
2
cos 2 , cos 2
1 1 1
sin 2 sin 2 , sin 2
2 2 2
1 1 1 1
sin 2 cos 2 cos 2 ,
2 2 2 2
x x
x x x
x x x x
dv
I e x dx u e x
dx
du
e x e x dx e v x
dx
dv
e x e x e x dx u e
= = =
= = =
= + =
}
}
}
2
2
sin 2
1 1 1 1
sin 2 cos 2 , cos 2
2 4 4 2
5 1 1
sin 2 cos 2
4 2 4
2 1
sin 2 cos 2
5 5
x x x
x x
x x
x
dx
du
e x e x I e v x
dx
I e x e x
I e x e x
=
= + = =
= +
= +


Thus,
( )
2 1 2 1 1
cos 1 sin 2 cos 2
2 5 5 2
1 1 1
sin 2 cos 2
5 10 2
x x x x
x x x
e x dx e x e x e C
e x e x e C
(
= + +
(

= + +
}




4 5 5 5
: 10, , , ,
2 8 32
x
5 5 5
: 5, , , ,
4 16 64
y

x: GP with 10 a = and
1
4
r =
y: GP with 5 a = and
1
4
r =
10
: 8
1
1
4
x S

= =
+
,
5
: 4
1
1
4
y S

= =
+

the ant will eventually end up at ( ) 8, 4 .

5 x > 2 or x s 2
When x > 2, |x 2| = x 2
2
2
( 2) 1
2 1 0
( 1) 2 0
( 1 2)( 1 2) 0
1 2 (N.A.) or 1 2
x x
x x
x
x x
x x
>
>
>
+ >
s > +



Hence, 1 2, 1 x x > + =


( 1) 1 1 x x + >
Let u = x + 1, 2 1 u u s B1
1 1 2 2,
1 1 0
u x x
u x x
= + > + >
= + = =










6

2
2
2
ln(cos ), ln(sin )
sin cos

cos sin
cos cos cos
cot
sin sin sin
x y
dx dy
d d
dy
dx
u u
u u
u u u u
u u u
u
u u u
= =
= =
| |
= = =
|
\ .

when , 1
4
dy
dx
t
u = =
1
ln
2
1
1
ln
2
y
x


The equation of tangent is ln2 y x = .
If this tangent meets the curve again,
ln(sin ) ln(cos ) ln 2
ln(sin 2 ) 0 sin 2 1
2 0
2 4 2
u u
u u
t t t
u u u
=
= =
| |
= = < <
|
\ .

The tangent will not meet the curve again as there is only one solution in
range 0
2
t
u < < i.e.
4
t
u = .

When x s 2, |x 2| = 2 x

No real solution except x = 1
OTHERWISE

sketch of
hence,

7 By Newtons Law of Cooling,
( ) 20

20
d
k
dt
d
k dt

=
=

} }

ln 20 kt C = +


Given: 0, 60 t = = A
10, 45 t = = B
From : ln40 C = , ln25 10k C = +

40
ln 10
25
k =

1 8
ln
10 5
k =

Also given: 0, ? t = =
20, 45 t = = C
From & :
( ) 2 1
1 8
ln 20 ln
10 5
t C = + ,

( ) 1
1 8
ln 25 20 ln
10 5
C = +

20 8
ln 2ln
25 5

= [Modulus can be removed as initial
temperature is higher than 45C, hence
20 0 > .]

( )
2
20 8
25 5

=

( )
2
8
25 20 84
5
= + = C



8 6, 1 a b = =

Asymptotes
1
2,
3
y x = =
Axial intersection : (0,1) , (1/6,0)

Asymptotes
1
2,
3
y x = =
Axial intersection : (0,1) , (0,-1), (1/6,0)

6 1 1
2
3 1 3 1
x
y
x x

= = +






I : A translation of 1 unit in direction of the positive x axis
II : A scaling parallel to the x axis with scale factor
1
3
units
III: A translation of 2 unit in direction of the positive y axis

OR
I : A scaling parallel to the x axis with scale factor
1
3
units
II : A translation of
1
3
unit in direction of the positive x axis
III: A translation of 2 unit in direction of the positive y axis

9
1
sin y x

=
2
1
1
dy
dx
x
=

.

2
1 1
dy
x
dx
=
( )
( )
( )
( )
2
2
2
2
2
2
2
3 2 2
2
3 2 2
3 2
2
3 2
diff. w.r.t ,
2
1 0
2 1
1 0
diff. w.r.t ,
1 2 0
1 3 0 ( )
x
d y dy x
x
dx dx
x
d y dy
x x
dx dx
x
d y d y d y dy
x x x
dx dx dx dx
d y d y dy
x x Shown
dx dx dx
| |

+ =
|
|

\ .
=
| |
+ + =
|
|
\ .
=

( )
( )
4 3 2
2
4 3 2
5 4 3
2
5 4 3
diff. w.r.t ,
1 5 4 0
diff. w.r.t ,
1 7 9 0
x
d y d y d y
x x
dx dx dx
x
d y d y d y
x x
dx dx dx
=
=


(0) 0, '(0) 1, ''(0) 0, '''(0) 1, ''''(0) 0, '''''(0) 9 f f f f f f = = = = = =
3 5
3
...
6 40
x x
y x = + + +

1
2
3 5
2 4
1
sin
1
3
...
6 40
3
1 ...
2 8
d
x
dx
x
d x x
x
dx
x x

| |
= + + +
|
|
\ .
= + + +


Using
1
2
x = ,
2 4
2
1 1
3
1 2 2
1 ...
2 8
1
1
2
| | | |
| |
\ . \ .
= + + +
| |

|
\ .








2 4
2
1 1
3
1 2 2
1 ...
2 8
1
1
2
| | | |
| |
\ . \ .
= + + +
| |

|
\ .

147
128


256
3
147
~

10
(a)

Area
1
2 2 4
0
1 4 x x dx =
}

Let
1
sin
2
x =
1
cos
2
dx

d
=
When 0, 0 x = = ;
1
,
4 6

x = = .
Substituting,
2 2
6
0
2 2
6
0
2 6
0
6
0
6
0
1 1
Area sin 1 sin cos
4 2
1
sin cos
8
1
sin 2
32
1
1 cos 4
64
1 1
sin 4
64 4
1 3
64 6 8

d
d
d
d

=
=
=
=
(
=
(

(
=
(

}
}
}
}



10
(b)


From G.C., approximate coordinates of the points of intersection of the

two curves are ( ) 1.36759, 0.65435 and ( ) 1.87156, 1.08307 .

( )
2
1.87156
1.36759
1
Volume ln 0.0766 (3 s.f.)
6
x
x x e dx = =
}







A

11
2
3
2
3
2
3
2
3 2 6
arg( 2)
arg[ ( 2 )]
arg arg( 2 )
arg( 2 )
iz
i z i
i z i
z i
t t
t
t
t
t
+ =
=
+ =
= =










5
4 6 12
2 2
5
12

3 3 3 2
sin 4.10 (3 sig fig)
3 2
NPQ
PQ
NQ
NQ
t t t
t
= + =
= + =
= =


Least value of | z 3 + i | is 4.10

Range of values of m for which there is exactly one complex number w
is m = 4.10, 3 2 m>




12
(i)


(ii)












(iii)

Asymptotes , 2 y x a x = + =

( )
2
2
4
1
2
dy a
dx
x
=


For stationary points, 0
dy
dx
=
( )
2
2
4
0 1
2
2 2
a
x
x a
=

=


when 2 2 , 2 5 x a y a = + = +
when 2 2 , 2 3 x a y a = =









6
t
4
t
P(0, 2)
Q(3, 1)
N

X


13
2
3
3
1 , 1, 0, 1
k
i
w w e k
t
= = =
2
3
3 3
3 3 3 3
3
( ) ( ) 0
( ) ( ) ( 1) ( )
1
k
i
z i z i
z i z i z i
z i
z i
z i
e
z i
t
+ + =
+ = =
+ | |
=
|
+
\ .
+
=
+

( ) ( )
( ) ( )
( )
( )
2
3
2
3
2 2
3 3
2 2
3 3
2
3 3 3
2
3 3 3
3 3 3
3 3 3
3
( 1 )
1
( 1 cos sin )

1 cos sin
1 1 2sin 2sin cos

1 2cos 1 2sin cos
2sin cos sin

2cos cos sin
tan
k
k
i
i
k k
k k
k k k
k k k
k k k
k k k
k
i e
z
e
i i
i
i i
i
i
i
t
t
t t
t t
t t t
t t t
t t t
t t t
t
+
=
+
+ +
=
+ +
(
+ +
(

=
+ +
+
=
+
=


ALTERNATIVE:
2
3
2
3
2
3 3
2
3 3
3 3
3 3
3
3
3
( 1 )
1
( 1 )
.
1
( )

(2 sin )

2cos
tan
k
k
k k
k k
k k
k k
i
i
i i
i i
i i
i i
k
k
k
i e
z
e
i e e
e e
i e e
e e
i i
t
t
t t
t t
t t
t t
t
t
t

+
=
+
+
=
+
+
=
+
=
=

( ) ( )
( )
( )
3 3
3 2 3 2
3
2
( ) ( ) 0
3 3 3 3 0
2 3 0
3 0
0, 3
z i z i
z z i z i z z i z i
z z
z z
z
+ + =
+ + + =
=
=
=

Since tan
3
z
t
= is negative (when k = 1) ,
tan 3 tan 3
3 3
t t
= = .




14(i)









(ii)






(iii)























(iv)







(v)
1
1 1
3 . 2
1 2
cos 107.5 (1 d.p)
11 9
BOC

| | | |
| |

| |
| |
\ . \ .
= =

Area of OBC A
( )
1
sin 107.5
2
OB OC =

( )
1
11 81sin 107.5
2
=

2
14.2 units =

2 2 3 1 2
1 4 6 7 4
3 4 6 3 4
t t
CP t t
t t
| | | | | |
| | |
= + = +
| | |
| | |

\ . \ . \ .
,
2
9
5
CB
| |
|
=
|
|

\ .

Since,
. 110
1 2 2
7 4 . 9
3 4 5
110
110
CB
CP
CB
t
t
t
=
| | | |
| |
+
| |
| |

\ . \ .
=



| 60t + 80 | = 110

60 80 110 or 60 80 110
1 1
or 3 ( 3.17)
2 6
t t
t
+ = + =
=



1
2 2
2
PA t
| |
|
=
|
|

\ .

1
2
2
AB
| |
|
=
|
|

\ .

Since
( )
2 PA t AB = , hence, P, A and B are collinear for all values of t.
[OR ( )
( )
1 2 PB t AB = , OR
( )
1
1
2
PB PA
t
| |
= +
|
\ .
]

OBC A and OPC A share the same base of OC. Since // OC BP , OBC A
and OPC A also has the same perpendicular height.
Area of OPC A = Area of
2
14.2 units . OBC A =








Anglo-Chinese Junior College
H2 Mathematics 9740
2008 JC 2 PRELIM PAPER 2 Solutions

1
Let
3 2
2 2
3 12 1
1
( 1)(3 1) 3 1
x x B Cx D
A
x
x x x
+ + +
+ +

+ +


Using any appropriate & reasonable method,
1, 4, 3, 2 A B C D = = = =

3 2
2 2
3 12 1 4 3 2
1
1
( 1)(3 1) 3 1
x x x
x
x x x
+ + +
= + +

+ +


( )
( ) ( )
3 2
2 2
2
2
2 1
2 1
3 12 1 4 3 2
1
1
( 1)(3 1) 3 1
4 1 6 2 1
1
1 2 3 1
3 1
3
1 2
4ln 1 ln 3 1 3 tan
2 3
1 3
1 2 3
4ln 1 ln 3 1 tan 3
2 3
x x x
dx dx
x
x x x
x
dx dx dx dx
x
x
x
x
x x x C
x x x x C

+ + +
= + +

+ +
= + + +

+
+
( | |
= + + + + +
( |
\ .
= + + + + +
} }
} } } }

2
2 2
0 x Ay By C + + + =
Sub(2.15, 0) ,
2
2.15 4.6225 C = =
Sub(1, 6.8) ,
2
1 6.8 6.8 0
46.24 6.8 3.6225
A B C
A B
+ + + =
+ =

Sub(0, 7) ,
49 7 0
49 7 4.6225
A B C
A B
+ + =
+ =


Solving, A = 0.6381827731 , B = 3.806922269

2 2
2 2
2 2
2 2
0.6382 3.8069 4.6225 0
0.6382( 5.9651 ) 4.6225 0
0.6382[( 2.9826) 8.8959] 4.6225 0
0.6382( 2.9826) 10.2999
x y y
x y y
x y
x y
+ =
+ =
+ =
+ =

When y = 2.9826,
2
10.2999 3.2093 x x = =
Widest part of tunnel is 6.4 cm (to 1 d.p.)
3
1 2 3 4
1 1 3 1
, , ,
2 2 8 4
u u u u = = = =

1
1
1 3
2
2 2
r
r
u
=
= =


2
1
1 1 4
2
2 2 4
r
r
u
=
= + =


3
1
11 5
2
8 8
r
r
u
=
= =


4
1
13 6
2
8 16
r
r
u
=
= =


conjecture for ( )
2
f
2
n
n
n
+
=
1 1
2 2
2 , i.e., 2 2
2 2
n n
r
r n n
r r
n n
u r

= =
+ +
= =


Proof:
Let ( ) P n be the statement:
1
2
2 2
2
n
r
n
r
n
r

=
+
=


To show ( ) P 1 is true: LHS =
1
1
1 2
2

=
RHS =
3 1
2
2 2
=

( ) P 1 is true.
Assume ( ) P k is true, i.e.
1
2
2 2
2
k
r
k
r
k
r

=
+
=

.
To show ( ) P 1 k + is true:
LHS = ( )
( )
1
1
1 1
2 2 1 2
k k
k r r
r r
r r k
+
+
= =
= + +


= ( )
( ) 1
2
2 1 2
2
k
k
k
k
+
+
+ +
= ( ) ( )
1
1
2 2 2 1
2
k
k k
+
+ + (


=
1
3
2
2
k
k
+
+
= RHS
Since P(1) is true and P(k) is true P(k+1) is true, hence by induction, ( ) P n is true for 1 n > .

4 Graphical Method:







Since the horizontal line y k = ,
f
, k R e cuts the graph of ( ) f y x = more than once, f is a not a
one-one function. Hence, f
-1
does not exist.
(Analytical Method):
( ) ( ) f 2 f 4 3 = = , f is a not a one-one function. Hence, f
-1
does not exist.

Largest | |
f
3, 5 D = .
Finding f
-1
, let
2
( 6 5) y x x = +

( )
-1
3 4
3 4 , 3 4 (N.A since 3 5)
f 3 4 , 0 4
x y
x y x y x
x x x
=
= + = s s
= + s s

( )
g
, R = + and ( )
h
, 2 D = . Since R
g
_
/
D
h
, hg does not exist.

hr exists if
r h
R D _ .
( )
h
, 2 D = and maximum ( )
r
, 2 R = .

x
y

2
5
6
Finding k,
2
ln 2 x
x e
=
=

Maximum value of . k e =

( )
hr
0, R =

5(i)















(ii)












(iii)







(iv)












(v)
8 0 8
0 0 0
10 6 4
EF
| | | | | |
| | |
= =
| | |
| | |
\ . \ . \ .
,
2 0 2
4 0 4
4 6 2
EG
| | | | | |
| | |
= =
| | |
| | |

\ . \ . \ .

Let n be the normal of Plane EFG.
8 2 2
0 4 8 3
4 2 4
| | | | | |
| | |
= =
| | |
| | |

\ . \ . \ .
n
Plane EFG:
2 0 2
. 3 0 . 3 24
4 6 4
2 3 4 24 (Shown). x y z
| | | | | |
| | |
= =
| | |
| | |
\ . \ . \ .
+ + =
r


Let u be the angle between Plane EFG and the horizontal ground.
Horizontal plane:
0
r. 0 0.
1
| |
|
=
|
|
\ .

1
0 2
0 . 3
1 4
cos
1 4 9 16
4
cos 42.0 . (1 d.p)
29
u
u

| | | |
| |
| |
| |
\ . \ .
=
+ +
| |
= =
|
\ .


Let h be the vertical distance from the drilled hole to the ground.
3 2
2 . 3 24
4
6 units.
h
h
| | | |
| |
=
| |
| |
\ . \ .
=

Point of intersection of line:
3 0
2 0
0 1

| | | |
| |
= +
| |
| |
\ . \ .
r and plane EFG:
2
. 3 24.
4
| |
|
=
|
|
\ .
r
Vertical wall partition: 2 8 x y + =
Plane EFG: 2 3 4 24 x y z + + =
Using GC,
2 1 0 8 1 0 0.5 0
Rref
2 3 4 24 0 1 1 8
( (
=
( (



Line GM:
0 0.5
8 1
0 1

| | | |
| |
= +
| |
| |
\ . \ .
r .









Vertical wall partition:
2
. 1 8.
0
| |
|
=
|
|
\ .
r
2 8 6
4 0 4
4 0 4
AG
| | | | | |
| | |
= =
| | |
| | |
\ . \ . \ .
or
6
4
0
AB
| |
|
=
|
|
\ .

Let the shortest distance be d.
2 6 2
1 4 1
0 4 0
8
3.58 units.
4 1 4 1 5
AG
d
| | | | | |
| | |

| | |
| | |
\ . \ . \ .
= = = =
+ +

6a)i)




ii)

b)
It is possible to use systematic sampling.
Random and stratified sampling would be awkward because the sampling frame is not fixed
OR
The exact number of customers is not known in advance.

The first 100 customers will not be representative of all supermarket customers midday and
late night shoppers will be unrepresented.
No.
Each household has different number of people, so the probability of being chosen is not equally
likely.
OR (give example)
A person in a household of one will stand more chance of being chosen than a person on a
household of four. Hence the method does not produce a random person.
7 To test H
0
: = 9.5 against
H
1
: = 9.5 at 5% level of significance
Given : Reject H
0
at 5% level of significance

Reject H
0
at 2.5% level of significance
Not necessarily true nor necessarily false

The critical region at 2.5% level of significance is smaller than that at 5% level so the test
statistic may lie inside or outside the critical region at 2.5% level of significance

OR

Reject H
0
at 5% level of significance
p-value < 0.05

If p-value = 0.01 < 0.025 < 0.05 , reject H
0
at 2.5% level

If p-value = 0.03 < 0.05 but 0.03 > 0.025 , do not reject H
0
at 2.5% level
8 Let X be the number of boys out of 500 babies.
X ~ B(500, 0.005)
Since n is large and p is small and np = 2.5 < 5
therefore X ~ Po( 2.5 ) approximately

P ( at most 495 of the babies are girls)
= ( ) 5 P X >
= ( ) 1 4 P X s
= 0.109
9i) No of ways = 4! 5!


ii)
= 2880

No. of ways =
6
3
3! 5! C
= 14400
10i)





10ii)





10iii)
Let A be the event that the orange cat is in the right cage.
Let B be the event that the black cat is in the right cage.
Required probability =
( )
( )
P
P( | )
P
C
C
C
A B
A B
B

= =
1 3
.
5 4
1
1
5

=
3
16
or 0.188
Required probability =
( ) ( ) P 1 P
C C
A B A B =
=
1 1 1 1
1 .
5 5 5 4
| |
+
|
\ .

= 0.65

Let X be the number of times out of 100 such that the black kitten and the orange kitten are
placed in the wrong cages.
X ~ B( 100, 0.65 )
But n is large and p is not too small and np = 65 > 5 and nq = 22.75 > 5
therefore X ~ N (65, 22.75) approximately

P(X = 60) = P(59.5 < X < 60.5) ( applying continuity correction)
= 0.0483

11i)






ii)

Let R be the mass of a randomly chosen red apple. R ~ N (75,
2
12.5 )
Let G be the mass of a randomly chosen green apple. G ~ N (55, 10.5
2
)
1 2 3
6469
2 ~ 15,
12
G G G R N
| |
+ +
|
\ .

Required probability =
( )
1 2 3
P 2 0 G G G R + + > = 0.741

P( 70) R < = 0.344578 = 0.345
Let X be the number of red apples out of 10 which are underweight.
X ~ B(10, 0.344578)
P( X < 2 ) = 0.0915378 = 0.0915

Let Y be the number of bags out of 20 which pass the quality test.
Y ~ B ( 20 , 0.0915378 )

Required probability = P(Y = 0) = 0.147
12a)i)














n
x
x

=
_
=
50
1230
= 24.6
( 20) (20 x 50) x x = = 1230 1000 = 230


( )
2
2
2
Unbiased estimate of population variance
( 20)
1
( 20)
49
1 230
1129
49 50
71
1.448979592 1.45 (3 )
49
x
x
n
sf
(

(
=
(

(
=
(

= = =













12a)
ii)

(b)

Test H
0
: 25 against
H
1
: = 25
Under H
0
, since n = 50 is large,

X ~
71
N 25,
(49)50
| |
|
\ .
approx. by CLT.
Using two-tailed Z test at 5% level,
p-value = 0.0188 < 0.05

Reject H
0
and conclude that there is sufficient evidence at 5% level of significance that there has
been a change in the mean of the distribution.

No, Since sample size is large, Central Limit Theorem applies.

A t-test should be used because the population variance is unknown and the sample size is small
13i)



ii)







iii)
Let T be the total number of deaths in the country in 5 days.
T ~ Po(15)
P(T = 9) = 0.0324

Let D be the number of deaths in the northern parts in 10 days.
D ~ Po(20)
Let D be the mean number of deaths over 50 periods of 10 days.
D ~
2
N 20,
5
| |
|
\ .
approx by Central Limit Theorem since n = 50 is large
Required probability =
( )
P 21 30 D s s = 0.0569

Let S be the number of deaths in the southern parts in 2n days.
S ~ Po(2n)
Since 2n > 10, S ~ N(2n, 2n) approximately

( ) 25 0.01 P S > >
( ) P 24.5 0.01 (applying continuity correction)
24.5 2
P 0.01
2
24.5 2
P 0.99
2
S
n
Z
n
n
Z
n
> >
| |
> >
|
\ .
| |
< <
|
\ .


24.5 2
2.326348
2
n
n

<

2 2.326348 2 24.5 0 n n + >
4.417831 n < or 2.772854 n >
7.688720 n >
least n = 8

14a)
(i)






(ii)
False
Method 1: The linear relation holds only for the sample. The population may not have a linear
relationship.
Method 2: Use a diagram




False
sample



(b)(i)








(ii)


(iii)




(iv)
r = 0 an absence of linear correlation between X and Y but there might be some curvilinear
relationship. Thus X an Y may be correlated.

Least square line of regression of y on t is y = 2.88112t + 1.65431
i.e y = 2.88 t + 1.65 ( to 3 sf)

ln x = 2.88112t + 1.65431
x = e
1.65431
e
-2.88112t


Hence x
0
= e
1.65431
= 5.23 (to 3 sf)
k = 2.88 (to 3 sf)

When t increases by 1, y decreases by 2.88.

x = 1.5, y = ln1.5, t =
88112 . 2
65431 . 1 5 . 1 ln


= 0.433
Use y on t because
(1) t is the independent variable and y is the dependent variable; or
(2) t is exact

r
2
= 0.9
2
= bd where b = 2.4
Hence d =
4 . 2
9 . 0
2
= 0.3375
Hence y = 0.3375u + k
Since
_
y = 0.074364,
_
u = 2.4
_
y + 10 = 2.4(-0.074364) + 10 = 9.82153
Hence k =
_
y 0.3375
_
u = 0.074364 0.3375(9.82153) = 3.39 (to 3 sf)

Hence the least squares line of regression of y on u is
y = 0.338u 3.39 (to 3sf)




2008 CJC H2 Maths Prelim Paper 1


1. Let x be the no. of toy cars of type A
Let y be the no. of toy cars of type B
Let z be the no. of toy cars of type C

2x + 3y + z = 100 (1)
x + 4y +3z = 150 (2)
x + 5y + 2z = 130 (3)

Using GC, x = 20, y = 10, z = 30

No. of toy cars of type A is 20
No. of toy cars of type B is 10
No. of toy cars of type C is 30

Total profit = 20($1) +10($2)+30($3) = $130
















2.
0 ) 5 x (
2 x
10 x
>
+


0
2 x
) 5 x )( 2 x ( 10 x
>
+
+

0
2 x
x 4 x
2
>
+
+

0
2 x
) 4 x ( x
<
+


x < - 2, or 0 < x < 4

For the inequality , 6 x
1 x
11 x
>
+

replace x above by 1 x
Hence, 2 1 x < or 4 1 x 0 < <
1 x < (n.a.) or 1 < x < 5
1 < x < 25






















3.
(i) Let the first three terms of the GP be
2
ar ar a , ,
.

( )( ) ( ) 1728
3 3 3 2
= = = ar r a ar ar a
[M1]
12 1728
3
= = ar

( ) 1
12

a
r =


When the third term is reduced by 2, the resulting AP is
( ) 2
2
ar ar a , ,


So
n n n n
T T T T =
+ 1 1


( ) ar ar a ar = 2
2


( ) 0 2 2
2
= + a ar ar


0
12
2 2
12
2
= + |
.
|

\
|
|
.
|

\
|
a
a
a
a
a


0 144 26
2
= + a a

So
18 = a
or
8 = a



3
2
= r

2
3
= r
(NA as series is convergent)

Therefore the three terms of the GP are 18, 12, 8.

(ii) For convergent series,
1 < r



54
3
1
18
3
2
1
18
1
= =

=
r
a
S


(iii) The AP has first term
18 = a

and common difference
6 18
3
2
18
1
= |
.
|

\
|
= = =

a ar T T d
n n


( ) | | d n a
n
S
n
1 2
2
+ =

( ) | | 1 6 36
2
= n
n

| | n
n
6 42
2
=

2
3 21 n n =


5S
n
> S
54 3 21 5
2
> ) ( n n

0 54 105 15
2
< + n n


441 6 559 0 . . < < n


n = 1,2,3,4,5,6



4. (i)

Since every horizontal line cuts the graph of f(x) at most once, therefore f is one-
one. Since f is one-one, hence
1
f exists.

(ii)
|
|
.
|

\
|
=
=
=

=

y
x
y
e
y
e
e
y
x
x
x
1
1 ln
1
1
1
1
1
1



























1 ,
1
1 ln ) (
1
>
|
.
|

\
|
=

x
x
x f
(iii) Since ) , 0 ( ) 0 , ( = . =
f g
D R , fg does not exist.
Since ) , 0 ( ) , 1 ( = c =
g f
D R , gf exists.
0 ,
1
1
1
1
1
) ( >

=
|
.
|

\
|

=

x
e e
g x gf
x x


(iv) k =1
0 , 1 ) (
1
1
1
s =
=
=

x x x g
y x
x y















5. (i) Using GC, the root o = 0.16744919 ~ 0.16745 (to 5 decimal places)
(ii) Using x
1
= 0, x
2
=
1
6
(x
3
1
+ 1) =
1
6
( [0]
3
+ 1) = 0.166 666 666 7
x
3
=
1
6
( [0.166 666 666 7]
3
+ 1) = 0.167 438 271 6
Using GC command:
1
6
( [Ans]
3
+ 1), we generate the rest of the x
i
values
x
4
= 0.167 449 038
x
5
= 0.167 449 189 (stabilised at 5 decimal places)
So the sequence converges to o ~ 0.16745
(iii) Since when n , x
n
a and also x
n + 1
o
Hence, the recurrence relation x
n + 1
=
1
6
(x
3
n
+ 1) tends to o =
1
6
( o
3
+ 1)
6 o = o
3
+ 1
o
3
6 o + 1 = 0
But this is just the equation where o is its root.
Hence the sequence can be used to estimate the root o.




6. (a)













For no real roots, 3 < m < 0

(b)(i) y = f (3x 1)
























y
x
0
2
1
(1,3) 2
4
4
y =
(x 4)
2
x
2
4

y
x
0
2
(1, 3)

7
(i) t
dt
dx
cos = t k
dt
dy
sin =
t k
dx
dy
tan =
(ii) For tangents parallel to y-axis, t =
2
,
2
t t

For tangents parallel to x-axis, t = t t , 0 ,

(iii) At k
dx
dy
t = = ,
4
t

Grad of normal =
k
1

2
1
4
sin + = + = k k x
t

2
4
cos
k
k y = =
t

Equation of normal:
|
.
|

\
|
=
2
1 1
2
k x
k
k
y
Since normal has a y-intercept of -1,
|
.
|

\
|
=
2
1
0
1
2
1 k
k
k

1 = k

(iv) When k = 1, equation of normal:
1
2
1
1
2
1
=
=
x y
x y









































(ii) y
2
= f (x)










(iii) y = f '( x )












y
x
0 1




y
x
0 1 2


At point P,
4
3
1 tan
sin cos
1 sin 1 cos
t
=
=
=
+ =
t
t
t t
t t

When ,
4
3t
= t
2
1
1 = x
2
1
= y
Intersection point P = |
.
|

\
|

2
1
,
2
1
1 (Ans.)





















8.
Solution: (a)(i) dx
x
x
x x dx x
} }

=

2
1 1
4 1
2
2 sin 2 sin
= c x x x + +
2 1
4 1
2
1
2 sin

(a)(ii)
( )
( )
dx
x
dx
x x
x
dx
x x
x
} } }
+
+


=
+
2
2
2 2
2 ) 1 (
1
3 2
2 2
2
1
3 2

=
c
x
x x + |
.
|

\
|
+ +

2
1
tan
2
1
3 2 ln
2
1
1 2



(b)
} }
+
=
+
u
u
u
d dx
x 1 tan
sec
1 4
2
2
2
2

=
}
u u d sec
= c + + u u tan sec ln
= c x x + + + 2 4 1 ln
2

(c)
} } }
+ =
4
2 2
1
2 2
4
1
2 2
a
a
dx a x dx x a dx x a
=
4
2
3
1
3
2
3 3
a
a
x a
x x
x a
(

+
(




=
3
65
5
3
4
2
3
+ a
a
































9.
(a)
i
b
i
a
i

+
+
= +
1
2
1
)
2
sin
2
(cos 2
t t


2
) 1 ( 2 ) 1 (
2 0
i b i a
i
+ +
= +




i
a
b b
a
i )
2
( )
2
( 2 0 + + = +

2
a
b = and 2
2
=
a
b
Solving a = -2, b = 1
(b) The other root is i
) 1 )( 1 ( ) )( 1 ( ) (
2 2 2
+ = = z z i z z z P
1
2 3
+ = z z z
(c) 81
4
= iz
i
i
i
i
z 81
81
4
= =
1 , 0 , 1 , 2 , 81 81
)
2
2 (
2
2
= = =
+
n e e e
i n
i n
i
t
t
t
t

1 , 0 , 1 , 2 , 3
)
8 2
(
= =
+
n e z
i
n t t


i
e z
)
8
7
(
1
3
t

= ,
i
e z
)
8
3
(
2
3
t

= ,
i
e z
)
8
(
3
3
t
= ,
i
e z
)
8
5
(
4
3
t
=













10.
Differentiating
2
x v y + = , with respect to x, x
dx
dv
dx
dy
2 + = .

2
2 1
x
y
x
dx
dy
+ =
2
2
2 1 2
x
x v
x x
dx
dv +
+ = + Correct substitution and simplification.
2 2
1 1
x
v
x
v
dx
dv
= = (Shown)

2
x
dx
v
dv
=
} }
=
2
x
dx
v
dv

c
x
c
x
v + = +

=

1
1
ln
1


When x = 1, y = 2, v = 1.

1
1
1
1 ln = = c
1
1
ln =
x
v
1
1

=
x
e v
1
1
2

=
x
e x y
1
1
2

+ =
x
e x y



























y


11
AQ = q a
= a + (1 )b a
= (1 )( b a)
= (1 )

AB

AQ //

AB and having a common point A.


Hence, the vector q is collinear with points A and B.

Given a =
\

|
.
|
|
|
1
1
0
and b =
\

|
.
|
|
|
4
2
6

Given also angle between OA and OQ is 60
|

OA | = 2

OQ = a + (1 )b =
\

|
.
|
|
|
1
1
0
+ (1 )
\

|
.
|
|
|
4
2
6
=
\

|
.
|
|
|
43
32
66

|

OQ | = (43)
2
+(32)
2
+(66)
2
= 54
2
108 + 56

Weve

OA .

OQ = |

OA ||

OQ | cos 60

\

|
.
|
|
|
1
1
0
.
\

|
.
|
|
|
43
32
66
= 2 54
2
108 + 56 (
1
2
)
4 = 108
2
216 + 112

4
2
= 108
2
216 + 112
0 = 108
2
216 + 96
(3 4)(3 2) = 0
=
4
3
or =
2
3

































12.
(a) 2 ) 2 ( = + z i z i
) 0 2 ( ) 2 0 ( i z i z + =










2
2
2
4
sin 2 ) 2 ( = = =
t
z


















12.
(b) (i)
OP
OQ
=
|z|
|1/z*|

=
|z|
|1/z|


2
2

x

2
= |z|
2

So OP : OQ = |z|
2
: 1

(ii) w = z +
1
z*

= 2(cos u + i sin u) +
1
2(cos u i sin u)

= 2(cos u + i sin u) +
1
2

1
(cos u i sin u)

cos u + i sin u
cos u + i sin u

= 2(cos u + i sin u) +
1
2

cos u + i sin u
(cos
2
u + sin
2
u)

=
5
2
(cos u + i sin u)
Re(w) =
5
2
cos u, Im(w) =
5
2
sin u






2008 CJC H2 Maths Prelim Paper 2


1a.








x x + ~ + 1 sin 1 Since x is small.
By standard Maclaurins Series expansion,
( )
2
2
1
! 2
1
2
1
2
1
2
1
1 1 1 x x x x
|
.
|

\
|

+ + ~ + = +
2
8
1
2
1
1 x x + ~ (Shown)

1b. Given 3
x
e
x ln a
= 0
3
x
= e
x ln a

3
x
= a
x

3 = a
Given that e
x
~ 1 + x +
x
2
2
+
x
3
3!

3
x
= e
x ln a

3
x
~ 1 + xln3 +
(xln3)
2
2
+
(xln3)
3
3!



1c.

1 ) 0 ( ) ( = = f x f y
2
1
2
sin 1
) 0 ( ' sin 1 2
2
2
=
+
= + =
y
x
f x
dx
dy
y
By implicit differentiation,
0
4
2
1
4 1
2
4 cos
) 0 ( " cos 4 2
2
2
2
2
2
2
2
=
|
.
|

\
|

=
|
.
|

\
|

= =
|
.
|

\
|
+
y
dx
dy
y x
f x
dx
dy
y
dx
y d
y
By implicit differentiation,
4
1
2
12 4 sin
) 0 ( " ' sin 4 12 2
2
2
2
3
3
2
2
3
3
2
=

|
.
|

\
|

= =
|
.
|

\
|
+ +
y
dx
y d
dx
dy
y
dx
dy
x
f x
dx
dy
dx
y d
dx
dy
y
dx
y d
y
By Maclaurins Expansion,
3
3 2
24
1
2
1
1
.....
! 3
) 0 ( " '
! 2
) 0 ( "
) 0 ( ' ) 0 (
x x
f
x
f
x xf f y
+ ~
+ + + + =




2.

Let P
n
be the statement
+
=
e
|
.
|

\
|
+
=
|
.
|

\
|
+
+

Z n
n r
r
n
r
,
3 2
3
ln
3 2
1 2
ln
1

When n = 1,

LHS of P
1
=
5
3
ln
3 ) 1 ( 2
1 ) 1 ( 2
ln =
|
|
.
|

\
|
+
+


RHS of P
1
=
5
3
ln
3 ) 1 ( 2
3
ln =
|
|
.
|

\
|
+
= LHS of P
1
P
1
is true.
Assume that P
k
is true for some
+
eZ k , i.e.
|
.
|

\
|
+
=
|
.
|

\
|
+
+

=
3 2
3
ln
3 2
1 2
ln
1
k r
r
k
r


We need to prove that P
k+1
is true, i.e.
|
|
.
|

\
|
+ +
=
|
.
|

\
|
+
+

+
=
3 ) 1 ( 2
3
ln
3 2
1 2
ln
1
1
k r
r
k
r


LHS of P
k+1

1
1
1
1
3 ) 1 ( 2
3
ln
3 ) 1 ( 2
3 2
3 2
3
ln
3 ) 1 ( 2
3 2
ln
3 2
3
ln
3 ) 1 ( 2
1 ) 1 ( 2
ln
3 2
1 2
ln
3 2
1 2
ln
+
=
+
=
=
(

+ +
=
(

+ +
+
|
.
|

\
|
+
=
|
|
.
|

\
|
+ +
+
+
|
.
|

\
|
+
=
|
|
.
|

\
|
+ +
+ +
+
|
.
|

\
|
+
+
=
|
.
|

\
|
+
+
=

k
k
r
k
r
P of RHS
k
k
k
k
k
k
k
k
k
r
r
r
r

.
1
true P true P
k k +
By Mathematical Induction, P
n
is true for all .
+
eZ n
|
.
|

\
|
+ =
|
|
.
|

\
|
+

|
|
.
|

\
|
+
+ + =
|
.
|

\
|
+
+

|
.
|

\
|
+
+
+ =
(

+
+

= = =
=
43
13
ln 3 ln 15
3 ) 5 ( 2
3
ln
3 ) 20 ( 2
3
ln 3 ln ) 1 6 20 (
3 2
1 2
ln
3 2
1 2
ln 3 ln
3 2
) 1 2 ( 3
ln
5
1
20
1
20
6
20
6
r r r
r
r
r
r
r
r
r

A = 15, B = 13, C = 43



3. (i) Since O lies on the line l, thus the plane t contains OA and parallel to (i + 4j 3k).
Thus a normal to the plane is
|
|
|
.
|

\
|
3
4
1
x
|
|
|
.
|

\
|
0
0
1
=
|
|
|
.
|

\
|

4
3
0

Eqn of the plane t is r .
|
|
|
.
|

\
|

4
3
0
= 0
(ii) Let u be the acute angle between OA and l.
cos u =
|
|
|
.
|

\
|
3
4
1
.
|
|
|
.
|

\
|
0
0
1
26
=
1
26

Shortest distance from A to l is AN.
AN = OA sin u = 1
1
26
= 5//26

(iii) Perpendicular distance from B to the plane t is
| n OB

. | = |
|
|
|
.
|

\
|
1
1
1
.
|
|
|
.
|

\
|
4
3
0
5
1
| =
7
5

(iv) Let BC intersect the plane at M and BM : MC = 1 : m

By ratio theorem, OM =
OC + mOB
1 + m
=
1
1 + m

|
|
|
.
|

\
|
+
+
+
m
m
m
3
3
6

M lies on the plane t, so OM .
|
|
|
.
|

\
|
4
3
0
= 0

1
1 + m
( 9 + 3m 12 + 4m) = 0
m= 3


4.
A = (1, 1) and B = (0, 2 2 2 )
Area R = dx x dx x
} }

+
0
1
2
0
1
) 2 2 (
= 0.448
Volume =
dy y dy y
} }


1
2 2
2 2
1
0
) 2 ) 2 (( t t

= 1.10


5. . (i) Only motorists in that particular residential estate are
interviewed. Hence it is a biased sample.

(ii) Systematic sampling

(iii) A better sampling method is stratified sampling.
The group is first divided into non-overlapping age groups. Then motorists are selected
randomly from each stratum (age group) to be interviewed. The number of people
interviewed in each stratum will be proportionate to the size of the stratum. In this way,
the sample is more representative of the population



6. (a)


No. of different 4-digit numbers = 3! x 3 = 18



(b) CUCUMBER

Case 1: all letters are different.
No. of 4-letter code words =
4
6
P = 360
Case 2: one pair of repeated letters.
No. of 4-letter code words = 240
! 2
! 4
x C x C
2
5
1
2
=
Case 3: two pairs of repeated letters.
No. of 4-letter code words = 6
! 2 ! 2
! 4
=
Total no. of 4-letter code words = 606


7.
(a) P[ the 3 guests are seated separately] =
12
5
! 9
x ! 6
3
7
=
P
[B1 each for 6!,
7
P
3
, 9! ]
(b) P[MRT/Late] =
P(
51
31
(0.1x0.01) x0.06) 5 . 0 ( (0.4x0.05)
(0.1x0.01) x0.06) 5 . 0 (
) =
+ +
+
=

Late
Late Car or Late Bus

[M1, B1 for numerator, A1 for final answer]
P[Late] = 0.4x0.05 + 0.5x0.06 + 0.1x0.01 = 0.051 [B1]

P[Late for second time on last day]
= P[Late once during the first 4 days and late on the last day] [M1or implied]
=
4
C
1
(0.051)(0.949)
3
(0.051) = 0.00889 [A1 for any two of
4
C
1,
(0.051),(0.949)
3
; A1
for final answer]


8. (i) Let W be the random variable no. of air bubbles in 1 randomly chosen plastic sheet.
Then W ~ Po(
1
2
)
P(W > 3) = 1 P(W s 2)
= 0.0143877
0.0144 (to 3 s.f.) (shown)



(ii) Let V be the random variable no. of air bubbles in 5 randomly chosen plastic sheets.
Then V ~ Po(
1
2
5) = Po(2.5)
Using GC, P(V = 1) = poissonpdf(2.5,1) = 0.2052
P(V = 2) = poissonpdf(2.5,2) = 0.2565 (highest probability)
P(V = 3) = poissonpdf(2.5,3) = 0.2138
Hence the most likely number of air bubbles is 2.

(iii) Let X be the random variable no. of plastic sheets with at least 3 air bubbles out of 15 plastic sheets.
Then X ~ B(15, 0.0144)
P(X > 2) = 1 P(X s 1)
= 1 binomcdf(15, 0.0144, 1)
= 0.0192244
0.0192

(iv) Let Y be the random variable no. of rejected crates out of 100 crates.
Then Y ~ B(100, 0.0192244)
Since n = 100 > 50 and np < 5
So Y ~ Po(1.92244) approximately
P(Y < 2) = P(Y s 1)
= poissoncdf(1.92244, 1)
= 0.42741
0.427



9. Let X be the waiting time on a randomly chosen day. X~ N(8,5)
Let Y be the journey time on a randomly chosen day. Y~N(11,4)
Let T be the total time taken on a randomly chosen day. T=X+Y~N(19,9)

(i) P(T>20) = 0.369 (to 3 sf)

(ii) Expected no. of days late in a month = 30 x 0.369 = 11.07 ~ 11 days.

(iii) Let c be the time taken.

P(T > c) < 0.05
P(T < c) > 0.95
c > 23.9346

Least value of c = 24 min

Thus, the latest time he can leave his house = 7:40am 24 min = 7:16am

(iv) Let T be the average time taken in a month.
T ~N(19,
30
9
)
P(15<T <20) = 0.966 (to 3sf)

10.
(i) 9 . 58 50
150
1335
= + = x
5 . 78 5134 . 78
150
) 1335 (
23580
149
1
2
2
= =
(
(

= s
(ii) H
0
: = 60
H
1
: < 60
= 5%
reject H
0
if p-value < 0.05
p-value = 0.0642 > 0.05
do not reject H
0
at the 5% level, as there is insufficient evidence to conclude that the mean speed of
the cars is less than 60km/h. Hence, it is not necessary to adjust the ERP rates.
(iii) It means that there is a 5% chance of wrong rejecting that the mean speed is 60km/h when it is in fact
60km/h.
(iv) As the sample size is large, it is not necessary to assume that the speed of the cars have a normal
distribution as Central Limit Theorem can be applied.
(i) (iv) From (ii), p-value = 0.0642
To reject null hypothesis, p-value <
> 0.0642
Smallest level of significance = 6.42%

11. (i)

Equation of regression line of x on t is: x = 0.26484t + 76.503
r = 0.877

(ii)

[Correct number of points with axis labeled B1]
[Straight line passing through one point with correct number of points on top and below B1]
(iii) When t = 130, x = 0.26484(130) + 76.503 = $110.93

The linear model is not very suitable since the estimated value differs from the actual value of $95
quite substantially.

(iv) For x = ae
bt
, where a, b > 0

Taking ln on both sides:
ln x = ln a + bt



Store t values in L1
Store x values in L2
Store ln x values in L3
Find reg. Line of ln x on t r = 0.9092 [A1]

For x = c + dt
2
where c, d > 0


Store t values in L1
Store x values in L2
Store t
2
values in L4
Find reg. Line of x on t
2
[M1]
r = 0.9607 [A1]

(v) The regression line to use is x = c + dt
2
where c, d > 0 since the r value is much closer to +1. So
University Zs model is more accurate.



Comparing Scatter Plot (Not Needed):

x = 0.26484t + 76.503 ln x = ln a + bt x = c + dt
2

r = 0.877 r = 0.9092 r = 0.9607



DUNMAN HIGH SCHOOL
H2 Mathematics
Year 6 Prelims P1 Solutions




Solution

1(i)
1
2
5
5
n
n n
S

=

( )
1
1
1 2
1
1
1
2 2
5 5
5 5
2
2 5
5
2
3 (1)
5
n n n
n n
n n
n
n
n
T S S

=
| | | |
=
| |
\ . \ .
= +
| |
=
|
\ .


1
1
2
3
2 5
5
2
3
5
n
n
n
n
T
T
+

| |
|
\ .
= =
| |
|
\ .
(constant)
series is a geometric series.


Alt:
1
2
3
5
n
n
T

| |
=
|
\ .
is in the form
1 n
n
T ar

= .
series is a geometric series.
1(ii)
1
2
5
n
n
T
r
T
+
= =
Since 1 r < , the series converges.


1
3
5
2
1
1
5
T
S
r

= = =




2 Let P
n
be the proposition
3
2
2
1
2
3
2
2
1
1
1
0
+

+
=
|
.
|

\
|
+

+
+
+

=
n n r r r
n
r

When n = 0, L.H.S. of P
0
=
6
5
3
2
2
1
1
1
= +


R.H.S. of P
0
=
6
5
3
2
2
1
2 =
P
0
is true.

Assume P
k
is true for some k > 0, i.e.
3
2
2
1
2
3
2
2
1
1
1
0
+

+
=
|
.
|

\
|
+

+
+
+

=
k k r r r
k
r


Consider P
k+1
,
R.H.S. of P
k+1
=
4
2
3
1
2
+

k k

L.H.S. of P
k+1
=

+
=
|
.
|

\
|
+

+
+
+
1
0
3
2
2
1
1
1
k
r
r r r


P of R.H.S.
4
2
3
1
2
4
2
3
1
2
1
3
2
2
1
2
4
2
3
1
2
1
3
2
2
1
1
1
1 + k
0
=
+

+
=
+

+
+
+
+
|
.
|

\
|
+

+
=
+

+
+
+
+
|
.
|

\
|
+

+
+
+
=

=
k k
k k k k k
k k k r r r
k
r

P
k
is true P
k + 1
is true

Since P
0
is true and P
k
is true P
k + 1
is true, then by induction,
3
2
2
1
2
3
2
2
1
1
1
0
+

+
=
|
.
|

\
|
+

+
+
+

=
n n r r r
n
r
, n > 0.
As n , 0
3
2
, 0
2
1

+ n n


1 0
5
6
7
6
1 1 2 1 1 2 1 1 2
1 2 3 1 2 3 1 2 3
(2 0 0)
r r
r r r r r r

= =
| |
|
\ .
| | | | | |
+ = + +
| | |
+ + + + + +
\ . \ . \ .
=
=



3 Let x, y, z be the price rate of electricity, gas and water respectively.


22 10 19 100
30 16 35 155
25 12 20 113
x y z
x y z
x y z
+ + =
+ + =
+ + =


augmented matrix =
22 10 19 100
30 16 35 155
25 12 20 113
| |
|
|
|
\ .

rref matrix =
1 0 0 2.2
0 1 0 2.5
0 0 1 1.4
| |
|
|
|
\ .




August's monthly utility bill = ( ) ( ) ( ) 28 2.2 19 2.5 21 1.4 $138.50 + + =



4










































5(a)
2 2
2 4 4 0 y x x + = ---- (1)

( )
2 2
2 2 4 y x x =
( )
2
2
2 1 1 4 y x
(
=



( )
2
2
2 1 2 y x =

( )
( )
2
2
2
1 1
2
y
x =


( )
1
asymptotes: 1
2
1 1 1 1
or
2 2 2 2
y x
y x y x
=
= =


2
0: (1) 4 0
2
x y
y
= =
=

From GC,











(b)
( )
2
2
8 2 0 y kx y + =
For real values of y,
discriminant =
( ) ( )( )
4
4 8 2 0 kx >

( )
4
64 kx >

4 2
4 4
4
4
2 2
x
k
>

2 2
x
k
> (shown)

Alt:

( )
4
2
8 0 kx >
( )
( )
( )
( )
2 2
8 8 0 kx kx + >

( ) ( )
2 2
2
2
2
8 or 8
2 2
(rej)
2 2
(shown)
kx kx
x
k
x
k
s >
>
>


6(i)






Since any horizontal line y = k cuts the graph of y = e
2x
at most once, f is one-one.

Let y = e
2x

y x
x y
y
x
ln
2
1
2 ln
e ln ln
2
=
=
=


8 8
Y
y = e
2x
y = k

x O
y
, ln
2
1
: f
1 -
x x 0 < x < 1











R
f
= (0, 1) _ D
g
= xe .
gf exists.

1 e 1 ) e ( ) e ( g ) ( gf
6 3 2 2
+ = + = =
x x x
x

7(a)
1
2 tan
e 0
y x
xy

=
Diff wrt x:

2
2
d d 1
2 0
d d 1
y y
y x
x x x
| |
+ =
|
+
\ .


2
2
1
d
1
d 1 2
y
y
x
x x
+
+
=



( )
( )( )
2 2
2
1 1
1 1 2
x y
x x
+ +
=
+

(b)(i)
1
ln
2
x
| |

|
\ .

( )
1
ln 1 2
2
x
(
=
(


( )
1
ln ln 1 2
2
x = +
( )
( ) ( )
2 3
2 2
1
ln 2
2 2 3
x x
x

= + + +

2 3
8
ln 2 2 2
3
x x x = + (shown)
1 2 1 x < s

1 1
2 2
x > >

1 1
2 2
x s <
(ii) ln2 2 y x =
8(a)
2 2
( 1) 3
(1 ) 1 ( ) ( ) ... 1
2 2
b
b b
ax b ax ax x x

+ = + + + = + +

Coeff of x:
1
1 ab a
b
= =
Coeff of x
2
:
2
2
( 1) 3 ( 1) 1 3
1 3
2 2 2 2
b b b b
a b b
b
| |
= = =
|
\ .

y = f(x)

y = f
1
(x)

x
O
y
y = ff
1
(x)


1
, 2
2
b a = =


2 1
2 2
2 2 2
2
(1 ) 3
(1 ...)(1 )
1 2
3
(1 ...)(1 ...)
2
3
1 ...
2
7
1 2
2
b
ax
x x x
x
x x x x
x x x x x
x x

+
= + + +

= + + + + + +
= + + + + + +
~ + +


2
2 1
2 1
2
1
1
2
1
2 2
2
1
...
2 2 2
1 1 1 2 2
...
2 4 8
1 1 9 2
(shown)
2 4 8
5 4
5 4cos
2 sin 2
(1 2 )(2 )
(1 2 )(1 )
( ) 1
x
x
x x
x x x
x x
x
x x
x x
x
x

| |

|
\ .
| |
+
|
\ .
+ + +
+

~
+ +
= + +
= + +
| |
= + +
|
\ .
=
~


9(a) Since
1
2 u = , 2
n
u = for all n
+
e , i.e.

2
2 2
2
2 1
a + +
=
+

6 6 a + =
6 36 a + =
30 a =
(b)(i) 0 a =
2
1 1
2
1
1
u u
u
u
+
=
+

For
2
u to be defined,

2
1 1
0 u u + > and
1
1 u =
( )
1 1
1 0 u u + >

1 1
1 or 0 u u s >
Since
1
1 u = ,
1 1
1 or 0 u u < >

(ii) As n ,
n
u o and
1 n
u o
+
,

2
1
o o
o
o
+
=
+

( )
2
1 o o o o + = +

( )
2
2 2
1 o o o o + = +
( ) ( )
2
2
1 1 0 o o o o + + =

( )( )
2
1 1 0 o o o o + + =




10(a) Given w* = z 2i, z = w* + 2i.
Therefore, |w|
2
= w* + 2i + 6.
Let w = i a b + .

( ) ( )
2 2
i 2i + 6
6 i 2
a b a b
a b
+ = +
= + +

Comparing real and imaginary parts,
2 b = 0 b = 2

( )( )
2
2
4 6
2 0
2 1 0
2 or 1(N.A.)
a a
a a
a a
a
+ = +
=
+ =
=

Therefore w = 2 + 2i.
10(b)



arg(a) =
4
t

a = 1 + i




11(a)
2
2
d
2 3
x
x
x x + +
}


2 2
2 2
2 1
2 1
2 2 2
d
2 3 2 3
2 2 2
d
2 3 ( 1) 2
2 1
ln( 2 3) tan
2 2
1
ln( 2 3) 2 tan
2
x
x
x x x x
x
x
x x x
x
x x C
x
x x C

+
=
+ + + +
+
=
+ + + +
+
= + + +
+
= + + +
}
}

(b)(i) 2sin x u =
d 2cos d x u u =

3

2
3
sin 3 sin 2
1
= = = =

u u x

6

2
1
sin 1 sin 2
1
= = = =

u u x

3
3 2 2
1
6
4 d 4 4sin 2cos d x x
t
t
u u u =
} }


3 2
6
2 1 sin 2cos d
t
t
u u u =
}


( )( )
2
1 1 4 1 1
0 or 1
2
1 5
2
o o o

= = =

=

Since 0.618 o ~ ,
1 5
2
o
+
=
Locus of (i)
Im(z)
Re(z)
Locus of (ii)
A
arg(a)
|a|

3 2
6
4cos d
t
t
u u =
}


3
6
2(cos2 1) d
t
t
u u = +
}


| |
3
6
sin 2 2
t
t
u u = +

3 2 3
( ) ( )
2 3 2 3
= + +

3
=
(b)(ii)
3 3
2
1 1
3
Area 4 d d x x x
x
=
} }


3
1

3ln
3
x
(
=


( 3ln 3 0)
3
=

3ln 3
3
=
(iii)
3
2
1
Volume d x y =
}

2
3
1
3
3
d
4.0 units
y
y
| |
=
|
|
\ .
=
}


12(i)
( )
2
d e e 1
d e
e
x x
x
x
y x x
x

= =

d
0 1
d
y
x
x
= =
Max y =
1
e

(ii)
1
e e
x
x
s

1
ln ln e ln
e
ln 1
ln 1, 0
x
x
x x
x x x
s
s
s >

(iii)
( ) ( )
2
2 2 2
d e e (1 ) 2
d
e e
x x
x x
y x x
x
+
= =
Concave upwards:

( )
2
2
2
d
0
d
2
0
e
2
x
y
x
x
x
>
+
>
>




y
x
(iv)









2
( 1) y k x =
a) k > 0
b) k < 0



DUNMAN HIGH SCHOOL
H2 Mathematics
Yr 6 Prelim P2 Solution



Solution:
1
( )
( )
2
2
2
14 14
4 2 4
3 3
2
2 0
3
x x x
x x
+ + = + +
= + + > e


3 2
2
3 2 2
2
2
2
6 15 21 28
3
2 7 15
6 15 21 28 3 (2 7 15)
0
2 7 15
14
6( 4 )
3
0
2 7 15
x x x
x
x x
x x x x x x
x x
x x
x x
+
>

+
>

+ +
>



Since
2
14
4
3
x x + + is always positive, we only require
2
2 7 15 0 x x > .
( )( ) 2 3 5 0 x x + >
3
or 5
2
x x < >
2
4 2 4 2i 32 32 8 = + =
arg( 4 2 4 2i ) =
4
t

i(2 )
3 3
4
4 2 4 2i 2 e
n
z
t
t
= =
2
i
3 12
2e
n
z
t t | |

|
\ .
= , n = 1, 0, 1.
Cube roots are:
3
i
4
2e
t

,
i
12
2e
t

,
7
i
12
2e
t
.
















The points
1 2 3
, and P P P lie on a circle of radius two centred at origin. The
P
2

2
2
2
Re(z)
Im(z)

P
3

P
1




vectors
1
2 3
, and OP OP OP are each separated by equal angles of
2
3
t
.

3









Surface area of stem = ( )
2
2 2 2 rx kx x k x t t t = =

2
2
d
d 2
where (shown)
2
x c
qx
t k x
p c
qx p
x k
= +
= =

p = 2q:
qx
x
q
t
x
=
2
2
d
d

3
2
2
3
3
3
3 3
3 3
d 2
d
d d
2
1
ln | 2 |
3
ln | 2 | 3 , where 3
2 e
2 e
qt
qt
x x
q
t x
x
x q t
x
x qt C
x qt D D C
x A
x A

| |
=
|
\ .
=

= +
= + =
=
=
} }


t = 0, x = 0:
0 = 2 Ae
0
A = 2
3 3
3 3
e 2 2
e 2 2
qt
qt
x
x

=
=


As t , 0 since 0 e
3
>

q
qt

3 3
2 0 2 = x .


4i)
Annual salary = ( )
10 1
36000 1.04 $51239

=
number)
4ii)
Amount saved in 1
st
year = ( )
1
36000 $18000
2
=

9
29
04 . 1
9
20
1 04 . 1
1000000
1 04 . 1
) 1 04 . 1 ( 18000
>
>
>


=
n
n
n
n
S

30
8 . 29
04 . 1 ln
9
29
ln
=
= >
n
n
4iii) 9 years = 108 months
108 = 21(5) + 3 n = 21

total amount = ( ) ( )
21
10000 2 25 20 3
2
+ + (


= $11155
5i)
1
l :
1 3
5 2 ,
12 2

| | | |
| |
+ e
| |
| |

\ . \ .
r = ,
2
l :
1 8
5 11 ,
12 6

| | | |
| |
+ e
| |
| |
\ . \ .
r =
Consider
3 8 34 2
2 11 34 17 2
2 6 17 1
| | | | | | | |
| | | |
= =
| | | |
| | | |

\ . \ . \ . \ .

A normal to
1
is
2
2
1
| |
|

|
|
\ .
.
1
2 1 2 2
2 5 2 2 4
1 12 1 1

| | | | | | | |
| | | |
= =
| | | |
| | | |
\ . \ . \ . \ .
: r r

5ii) Method 1
Line passing through Q and perpendicular to
1
,
4 2
: 0 2 ,
8 1
l o o
| | | |
| |
= + e
| |
| |
\ . \ .
r
Since R lies on l,
4 2
2
8
OR
o
o
o

+ | |
|
=
|
|
+
\ .
for some o e
At point of intersection of l and
1
, R
4 2 2
2 2 4
8 1
o
o
o
+ | | | |
| |
=
| |
| |
+
\ . \ .

4
3
8 4 4 8 4 o o o
o
+ + + + =
=

4
3
8
3
20
3
OR
| |
|
=
|
|
\ .
(shown)
Method 2







Q
R



RQ = Projection of PQ

onto the normal


=
1 1
1 1
PQ
| |
|
|
\ .
n n
n n


3 2 2
5 2 2
4 1 1
8
3
8
3
4
3
4 4 1 4 4 1
2
4
2
3
1
| | | | | |
| | |

| | |
| | |

\ . \ . \ .

| |
|
|
=
|
+ + + +
|
|
\ .
| | | |
| | | |
= =
|
| |
\ .
| |
\ . \ .

8 4
3 3
8 8
3 3
20 4
3 3
4
0
8
OR OQ RQ

| | | | | |
| | |
= = =
| | |
| | |
\ . \ . \ .
(shown)
5iii) 2 possible scenarios









1
: 2 2 4 x y z + =
2
: 2 5 x ay bz + + =
3
: 3 7 x y z =

Direction vector of line of intersection of
1
&
3

=
1 2 5
3 2 3
1 1 4
| | | | | |
| | |
=
| | |
| | |

\ . \ . \ .


5 2
3 0 10 3 4 0
4
3 4 10
a a b
b
a b
| | | |
| |
= + =
| |
| |
\ . \ .
+ =

6 No. It is not possible to obtain the sampling frame, as list of all readers who
purchased magazines is not available.

Method is unsatisfactory as sample is not random but depends on the readers
responses, e.g. readers who want the token (or have the time) are more likely to
respond.







The publisher decides on the strata, e.g. based on 4 different age groups, then
determines the number of readers to be surveyed from each age group.
The publisher then sends interviewers to newsstands to interview the required
number of readers from each age group.
7i) YY E E S T R D A
No. of arrangements = 20160
! 2
! 8
=
7ii) YY S T R D A

No. of arrangements = 15120
2
7
! 6 =
|
|
.
|

\
|

or
No. of arrangements = 15120 ! 7
! 2
! 8
=
8 Let X be no. of accidents involving cyclists
X ~ B (50, p)

50p (1 p ) = 2.82
p = 0.06 or 0.94 (rej as p < 0.5)

x P(X = x)
2 0.2262
3 0.231
4 0.1732 most probable number is 3

p = 0.2
P (10 < X < 20) = P (X s 19) P(X s 10)
= 0.416
9 Let X be weight of a randomly chosen tomato
9955 . 899 )
50
1581
94091 (
49
1
62 . 231 200
50
1581
2
2
= =
= + =
s
x


H
0
: = 220
H
1
: > 220

One-tail test at 5% sig level
Reject H
0
if p-value < 0.05

Since n = 50 is large, under H
0

)
50
9955 . 899
, 220 ( ~ N X
Carry out z test: pvalue = 0.00308

Since p-value < 0.05, we reject H
0
and conclude that there is sufficient evidence
at the 5% sig level that the manufacturers claim that the mean weight will
increase is correct.

The test remains valid. As sample size of 50 is large, by central limit theorem,
the distribution of the sample mean is still normal.
The p-value is the probability of obtaining a sample mean as extreme as 231.62
if the population mean is 220 g.
10i) X ~ N(800. 250) Y ~ N(600. 200)

P(X > 800) P(Y < 600) =
4
1
2
1
2
1
=
10ii)
) 1550 , 0 ( N ~ ) ( ) (
4 3 2 1 3 2 1
Y Y Y Y X X X + + + + +
102 . 0
) 50 ) ( ) (( P
4 3 2 1 3 2 1
=
> + + + + + Y Y Y Y X X X

10iii) The distribution of the weight of durian cakes is independent of the distribution
of the weight of mango cakes.
11i) P(drawing 2 cards each scoring more than 5 points)
=
32 20
2 1
52
3
| || |
| |
\ .\ .
| |
|
\ .
=
9920 496
0.449
22100 1105
= = or
=
1105
496
50
31
51
32
52
20
1
3
=
|
|
.
|

\
|

11ii) P(all three cards are of different suits)
=
4
3
| |
|
\ .
3
13
1
52
3
| |
|
\ .
| |
|
\ .
=
169
425
= 0.398 or
=
4
3
| |
|
\ .
13 13 13
3!
52 51 50
= 0.398 or
=
425
169
50
26
51
39
52
52
=
11iii) P(total score of 3 cards is more than 28 points given they are of different suits)
=
suits) different of are P(cards
suits) diff of are cards and points 28 score P(ttl >

=
scores are 10, 10, 9 scores are 10, 10, 10
P + P
and of diff suits and of diff suits
169
425
| | | |
| |
\ . \ .

=
4 3 4
4 4 1 4 4 4
3! + 3!
3 1 52 51 50 3 52 51 50
169
425
| || | | |
| | | |

| | | | |
\ . \ .
\ .\ . \ .
=
48 64 112
+
112
5525 5525 5525
0.0510
169 169
2197
425 425
= = = or

=
2197
112
425
169

! 2
! 3
50
4
51
12
52
16

50
8
51
12
52
16
=
|
.
|

\
|
+
|
.
|

\
|


Method 2
P(total score of 3 cards is more than 28 points given they are of different suits)
=
2197
112
1
13
3
4

1
1
1
4
! 3
3
4

1
4
3
4
3
2 3
=
|
|
.
|

\
|
|
|
.
|

\
|
|
|
.
|

\
|
|
|
.
|

\
|

|
|
.
|

\
|
+
|
|
.
|

\
|
|
|
.
|

\
|

12i) Let X be no. of larvae in a sample of 30 ml of water
X ~ P
o
(6)
P(X > 4) = 1 P(X s 3) = 0.849
12ii) Let Y be no. of vials with 30 ml of water and at least 4 larvae each
Y ~ B (50, 0.84880)

Since n =50 is large, np = 42.4>5, n(1 p)=7.56>5
Y ~ N(42.44, 6.4169) approx
P(Y > 40) = P(Y > 40.5) (continuity correction)
= 0.778
12iii) Since n = 50 is large, by Central Limit Theorem
approx )
50
6
, 6 ( N ~ X
00195 . 0 ) 5 P( = < X

Method 2
Let S be no. of larvae in 50 vials
S ~ P
o
(300)
Since = 300 > 10, S ~ N (300, 300) approx

00177 . 0
) 5 . 249 P(
) 250 P( ) 5 P(
=
< =
< = <
S
S X


13i)







The scatter diagram shows a strong positive linear correlation between x and y.

13ii) r = 0.955

0.2 0.4 0.6 0.8 1.0 x
y
1.0
0.8
0.6
0.4
0.2
a x a
b
y
ax
x b
y
x b
ax
y
1 1 1
1
+
|
.
|

\
|
=
+
=
+
=

relationship between
y
1
and
x
1
is linear
13iii)
34972 . 0 268755 . 0
30124 . 1 76850 . 0
1
76850272 . 0
1
268755 . 0
1
= =
= =
+
|
.
|

\
|
=
b
a
b
a
a
x y

13iv)
0704 . 0
76850272 . 0
02 . 0
1
268755 . 0
1
=
+
|
.
|

\
|
=
y
y

The calculated growth rate is not reliable as it lies outside the data range and the
relationship between
y
1
and
x
1
may not be linear outside the data range.


HCI 2008 Prelim Paper 1 Solution

Qn Solution
1
Let x, y and z be the no of 10 cent, 20 cents and 50 cents coins respectively.
The 3 equations are 10 20 50 1500 x y z + + = , x y z = +
50 1500
2
x y z + + | |
=
|
\ .

Aug matrix A =
10 20 50 1500
1 1 1 0
1 1 1 60
| |
|

|
|
\ .
rref A =
1 0 0 30
0 1 0 10
0 0 1 20
| |
|
|
|
\ .
30, 10, 20 x y z = = =
2
2
4 3 4 x x
2
(2 1) 2 x
2 2
2
( 1)(4 3 4 )
0
( 5)
x x x
x

>

2
2
1
0
5
x
x

s

2
(4 3 4 ) 0 x x < for all xe
5 1 x < s or 1 5 x s < .
3 Volume of water V
( )( ) ( )
2 2
1 1
3 9 9
3 3
r h t t =
( ) ( )
2
1 1
27 9 9
3 3
h h t t
(
=
(


( )
3
27 9
27
h
t
t =
( )
2
9 d
d 9
h V
h
t
=
d d d
d d d
h h V
t V t
=
( )
2
9 d
.
d
9
V
t
h t
=


Given
3 1
d
2 cm s when 6 cm
d
V
h
t

= =
( )
( )
1
2
d 9 2
2 cms
d
3
h
t t
t

= =
4i
( )
2
2 2
2
1
(2 1)
4
1
(2) (2 1)(2 )
4
x
x x
x
d
e x
dx
e x e
xe

( = +

=

4ii
2 2
2
2 2
2 2
1
(2 1)
4
ln(2 -1)
1 1
(2 1) ln(2 1)
4 2
1 1
(2 1) ln(2 1)
4 4
x x
x
x x
x x
xe dx e x
xe x dx
e x x e dx
e x x e C
=
=
= +
}
}
}

5
( )
( ) i 3 3 i i3
4 i 4 1 e e e z
t u t u +
+ = =
( )
3 3
4 i 4 1 z z + =
3 3
i4 4 i z z + = +
3
4 i
z i
1 4i
+
= =
+
i 2
3 2
z e , 1, 0,1
k
k
t
t
| |
+
|
\ .
= =
i
2
1: e i k z
t | |

|
\ .
= = =
i
6
3 1
0: e + i
2 2
k z
t | |
|
\ .
= = =

5
i
6
3 1
1: e + i
2 2
k z
t | |
|
\ .
= = =

6
( ) ( )
1
1 1
1
n
n n x
x x
x
| |
= +
|
+
\ .

=
( )
( )
( )
( ) ( )
( )
2
2
1
1 ( ) ...
2
1
1 ( ) ...
2
n n
n x x
n n
n x x

+ + +

+ + +
| |
|
\ .
| |
|
\ .

=
2 2
1 2 2 nx n x +
Expansion is valid if
{ }
: 1 x x e < .
Let x =
1
20
, n =
1
4
& sub into above expansion
1
2 2
4
19 1 1 1 1
1 2 2
21 4 20 4 20
| | | || | | | | |
~ +
| | | | |
\ . \ .\ . \ . \ .
=
3121
3200

7
50
dx
kx
dt
=

1
50
dx dt
kx
=

} }

1
ln 50 kx t C
k
= +

ln 50 kx kt B = +
When 0, 0, t x = = 50 A=
When
1
200,
20
x k = = ,
20
200 20 50 50
t
e
| |
=
|
\ .

4.46 t h =
20
dx x
dt
=
50
kt B
kx e
+
=
50
kt
kx Ae

=
( )
1
50
kt
x Ae
k

=


1 1
20
dx dt
x
=
} }
ln
20
t
x C = +
When 0, 80 ln80 t x C = = =
When 20, x =
ln 20 ln80
20
t
= +

27.7 t h =
8

Any line y k = ( 1 k < ) cuts the graph at 2
points.
f is not one-to-one.
Therefore, f does not have an inverse. 3 b =
Let
(3 ) x
y e

= 3 ln( ) x y = +
1
f : 3 ln( ), 1 x x x

+ s
For
1
f g

to exist, ( | 1
g
f
R D , 1

_ =
( |
g
R , a = Therefore, 1 a s
Largest 1 a = and
| ) 1
f g
R 3,

=

9a
Let P
n
be ( )
2
2 1 4
n
u n = + for all n
+
e
P
1
:
1
u = 5 & ( )
2
1
2 1 1 4 5 u = + = P
1
is true
Assume that

P
k
is true for some n
+
e
We want to show that P
k+1
is true
( )
( )
( )
2
1
2
2
8 8 2 1 4 8 8
4 12 9 4
2 1 1 4
k k
u u k k k
k k
k
+
= + + = + + +
= + +
= + + (


P
k+1
is true
Since P
1
is true & P
k
is true P
k+1
is true ,
P
n
is true for all n
+
e .
( ) ( )( )
2
2 1 4 2 3 2 1
n
u n n n = + = +
b ( ) ( )
( )( ) ( )( )
( )( )( )
1
1 1
2 3 2 5 2 1 2 3
4
4
2 1 2 3 2 5
r
f r f r
r r r r
u
r r r

=
+ + + +

= =
+ + +

( ) ( ) { }
1
1
4
r
u f r f r =
1 1
........
3 5 7 5 7 9
n
S = + +

to n terms
=
( ) ( ) { }
1
1
1
4
n
r
f r f r
=


=
( ) ( )
( ) ( )
( ) ( )
( ) ( )
( ) ( )
1 0
2 1
3 2
1
4 ............
1 2
1
f f
f f
f f
f n f n
f n f n


+


+

`


+


+
)
=
( ) ( ) { }
1
0
4
f n f

=
( )( )
1 1 1
4 15 2 3 2 5 n n

`
+ +

)

As n ,
( )( )
1
2 3 2 5 n n + +
0
1
lim
60
n
n
S

=

10i 2
1
sin sin cos
2
dy
y
d
u u u
u
= =
2
1
4 4
2
0 0
2
4
0
1
sin
2
sin cos
1 2 cos
1
sin
2
y
dy d
y
d
=

=
} }
}
t
t
u
u u u
u
u u

4
0
4
0
1 1 cos 2
2 2
1 sin 2
2 2 2
1 1
4 2 2 2
d
t
t
u
u
u
u
t

=
(
=
(

(
=
(

}

Ii
When
4 4
1 1 1
, ( )
4 2 2
x y x = = = =

When
2
2
1
1 1
2
,
2 1 1 1 4 2
x
x y
x
= = = =
+ +
.
Thus two curves intersect at the point
1 1
,
4 2
| |
|
\ .
.(Verified)

1
1 1 5
1
4 4 4
1
0
1
4
4
4 1 1 4 9 2 2
Area
1 2 5 4 2 5 40 16 2 2
y
y dy dy y
y
(
(
= + = + = +
(
(



} }
t t

Alternatively
| |
1 2
1
4
2
1
2
0
2
1
1
5
2
1
2
0
2
1 1
2 2
0
0
2 2
1
2 1
1 1 1
1
5 2 2(2 1)
4 1 1 1
1
5 2 4
( )
2
x
x dx dx
x
x dx
x
x dx
x
=
+
( =

+
= +
+
} }
}
}

1
1
2
0
4 1 1 2
tan 2
5 4 20 2 2 2
4 9 2 2
5 40 16
x
t

(
= + +

= +

10
iii
Volume
1 1
1
4 2
1
0
4
1 2
fnInt(( / (1 2 ), , 0, 0.25)
fnInt( ^ (0.5), , 0.25,1)
[0.631620]
1.984
y
dy y dy
y
x x x
x x
t t
t
t
= +

+ (
=
(

=
=
} }

11i




2
1
2
x t
dx
t
dt
=
=

ln(2 )
1
2
y t
dy
dt t
=
=


1
2 (2 )
dy
dx t t
=


ii Equation of normal at ( 1, ln2) :
2
1 1
0
x t
t
= =
=
,
dy
dx
is undefined
Therefore, equation of normal: ln2 y =
Equation of tangent at ( 5, 2ln2) :
2
5 1
2
x t
t
= =
=
,
1 1
2 (2 ) 16
dy
dx t t
= =


2ln 2 1
5 16
5
2ln 2
16 16
y
x
x
y

=
+
= +

iii
5
ln(2) 2ln 2
16 16
x
= +

16ln2 5 x =
(16ln2 5, ln2) P =
iv
Let the angle u be the acute angle between normal and tangent.
1
tan 0.0624
16
u u = =

12i
( 1) a + b + c = 0 = 1
(b a) + (c a) = 0
0
1
where
AB AC
AB k AC k

+ =
= =

A, B, C are collinear
ii
p = 4a - 3b
4a = p + 3b
a = (p + 3b) / 4

Alternatively
4 3 3 3 a p a a b a b PA= = + = +
A divides PB in the ratio 3 : 1
P lies on BA produced with ratio
3
4
PA
PB
=

4 3 4 4 b p b a b a b PB = = + = +
P lies on BA produced with ratio
3
4
PA
PB
=

bi
( )
1
1 1
1 13 42
7 2
21 21 21
10 4
a r n
d
n
| | | |

| |
= = =
| |
| |

\ . \ .

Alternatively
Take a point in
1
[ , say C ( 13,0,0) which satisfies
1
2 13
4
r
| |
|
=
|
|

\ .


12
7
10
AC OC OA
| |
|
= =
|
|
\ .

Proj AC on
1
2
4
| |
|
|
|

\ .
=
12 1
1
7 2
21
10 4
n
n
AC
| | | |
| |
=
| |
| |

\ . \ .

2 21 =

Alternatively
Using a long method
Vector equation of a line through A and parallel to
1
2
4
| |
|
|
|

\ .
:
1 1
7 2
10 4
r
| | | |
| |
= + e
| |
| |

\ . \ .
R

Let C be the foot of the perpendicular from A to
1
[
Then
1
7 2
10 4
OC

+ | |
|
= +
|
|

\ .

C lies on
1
[ :
1 1
7 2 2 13
10 4 4

+ | | | |
| |
+ =
| |
| |

\ . \ .
= 2
1
7 2
10 4
OC

+ | |
|
= +
|
|

\ .
=
1
3
2
| |
|
|
|

\ .
;
1 1
7 2 7
10 4 10
AC

+ | |
|
= +
|
|
+
\ .

d = 84 2 21 AC = =
bii
Since d is positive, the angle between ( )
1
a r & n is acute
( )
1 1 3
1
2 7 2 2 21 2 1
21
10 4 6
n
d
n
OB OA
| | | | | |
| | |
= = =
| | |
| | |

\ . \ . \ .

Alternatively
Using a long method
Vector equation of a line through A and parallel to
1
2
4
| |
|
|
|

\ .
:
1 1
7 2
10 4
r
| | | |
| |
= + e
| |
| |

\ . \ .
R

Let C be the foot of the perpendicular from A to
1
[
Then
1
7 2
10 4
OC

+ | |
|
= +
|
|

\ .

C lies on
1
[ :
1 1
7 2 2 13
10 4 4

+ | | | |
| |
+ =
| |
| |

\ . \ .
= 2
1
7 2
10 4
OC

+ | |
|
= +
|
|

\ .
=
1
3
2
| |
|
|
|

\ .

By ratio theorem :
( )
1
2
2
OC OA OB OB OC OA = + =

3
1
6
OB
| |
|
=
|
|
\ .


iii
1
[ : x + 2y 4z = 13 . (1)
2
[ : x + 3y + 3z = 8 . (2)
By G.C. solve equations (1) & (2)
The vector equation of the line of intersection is
l

:
55 18
21 7
0 1
r
| | | |
| |
= +
| |
| |
\ . \ .
where e . or
1 18
0 7
3 1
r
| | | |
| |
= +
| |
| |

\ . \ .
etc
iv Since B and l lie on the image plane of
2
[ so the equation of image plane is
3 18 55 3
1 7 21 1
6 1 0 6
r
( | | | | | | | |
( | | | |
= + +
( | | | |
| | | |
(
\ . \ . \ . \ .

3 18 29
1 7 10
6 1 3
r
| | | | | |
| | |
= + +
| | |
| | |

\ . \ . \ .
where and e

Alternatively
Using
55 3 29
21 1 2 10
0 6 3
( | | | | | |
( | | |
=
( | | |
| | |
(
\ . \ . \ .
as a dir vector
n =
29 18 31
10 7 83
3 1 23
| | | | | |
| | |
=
| | |
| | |
\ . \ . \ .

31 3 31
83 1 83 38
23 6 23
r
| | | | | |
| | |
= =
| | |
| | |
\ . \ . \ .



HCI 2008 Prelim Paper 2 Solution

Qn Solution
1
1
2
2 tan
3
x
y
x

| |
=
|
|
+
\ .

2
2 2
2 2( 3) 2
2 1/ 1
( 3) ( 3)
dy x x x
dx x x
| |
| | | |
+
= + |
| |
| |
|
+ +
\ . \ .
\ .


2
2
2 3
dy
dx x x
=
+ +

( )
2
2 3 2
dy
x x
dx
+ + =
( )
2
2
2
2 3 2( 1) 0
d y dy
x x x
dx dx
+ + + + =
( )
3 2
2
3 2
2 3 4( 1) 2 0
d y d y dy
x x x
dx dx dx
+ + + + + =
When 0, x =

2 3
2 3
2 4 4
0, , ,
3 9 27
dy d y d y
y
dx dx dx
= = = =
2 3
2 2 2
3 9 81
y x x x ~ + (up term in
3
x )
2
2
1 1 1 2 4 2
2 2 3 9 27 2 3
dy
x x
dx x x
| |
= ~ +
|
+ + \ .

=
2
1 2 1
3 9 27
x x +
2i

2ii

2
iii

3i Amount owed before interest is added at end of nth year
1
12(1000)
n
u

= (as $12000 is paid for 1 yr)
Thus amt owed with interested added
1
1.03( 12000)
n n
u u

=
a
1
1
1 a
a
(a + 1, 4a)
(1 a, 0)
3ii
1
2
2 2
2 1
(200000 12000)1.03
((200000 12000)1.03 12000)1.03
200000(1.03 ) 12000(1.03) 12000(1.03 )

200000(1.03 ) 12000(1.03 1.03 ... 1.03 )
1.03(1.03 1)
1.03 (200000) 12000
0.03
1.03 (200000
n n
n
n
n
n
u
u
u

=
=
=
= + + +
(
=
(

= ) 412000(1.03 ) 412000
412000 212000(1.03 )
n
n
+
=

3
iii
0 when 23
n
u n < =
(Use GC, Sequence mode) or
412000 212000(1.03 ) 0
212000(1.03 ) 412000
412
1.03
212
412
lg
212
22.48
lg1.03
least 23
n
n
n
n
n
<
>
>
> =
=

Needs 23 years
3iv
Total interest paid
22
16
0.03
0.03(277247.491)
8317.42
r
r
u
=
=
=
=


15
15
412000 212000(1.03 ) 81710.90768 u = =
Penalty =
15
0.05 4085.55 u =
As total interest paid is greater than penalty,
its to Johns benefit to terminate his loan early.
4i 1 z =
( ) arg 2 3
4 3
z
t t | | | |
=
| |
\ . \ .

3
2
2
t
t +
2
t
=
4ii
cos i sin
2 2
z
t t
= +
cos isin
2 2
n
n n
z
t t
= +
{ }
{ }
cos 0 2 1,
2
or 2 1,
n
n m m
n m m
t
= = + e
= e

4
iii
( )
3 i
arg arg 3 i
2 2
w
| |
+ =
|
|
\ .

3 i
arg
2 2 6
w
t
| | | |
+ = | |
| |
\ . \ .


4
iv
( )
3 i 2 3 i 2 u u + = =
3 i
1
2 2
+ =
Greatest
3 i
1 4 5
2 2
u + = + =
5i
Quota Sampling
5ii
A list, in alphabetical order of the male and female employees can be generated separately. The list of
male employees will be numbered from 1 to 580 in that order while the list of female employees will be
numbered from 1 to 420 in that order. A number between 1 to 20, inclusive is then selected at random. The
corresponding employee and every 20
th
employee thereafter will be included in the sample for interview.
This method of selection is done for both the list of male and female employees.
5iii
In (ii), the sample selected would be random whereas in (i), the sample is non-random as the interviewers
were free to choose who they wanted to interview thus creating biasness.
6
0
H : 55 = vs
1
H : 55 >
1014
67.6
15
x
x
n
= = =


( )
2
2
2 2
1 1 (1014)
83568 1072.9714
1 15 1 15
x
x
s x
n n
| |
| |
|
= = =
|
|
|
\ .
\ .


Under
0
, T t (14) H . Using GC, select t- test, p - value = 0.0792
Since p - value = 0.0792 < 0.10, we reject H
0
and
conclude that there is sufficient evidence at the 10% level of significance that the promotional campaign
was successful in encouraging customers to spend more money at the store.
7
General comments for Q7
- There is a general lack of explanation on the approach to the qn. When solving a qn by considering
cases, candidates should state the cases clearly.
- Many students just wrote a 'string of numbers' without explanation.
- Solution only has all numerical computations without any words; not even the phrase 'number of ways".
- Use of diagrams is lacking. Diagrams are helpful for marker's understanding (if you are not good in
words)
7
No. of ways the committee can be formed
=
4 10 4 10
3 5 4 4
C C C C + = 1218
7i No. of ways the committee can be seated without restriction = (8 1)! = 5040
7ii
No. of ways the committee can be seated such that no two girls are seated next to each other = 4! (5)(4)(3)
= 1440

No. of ways the committee can be seated when there are two absentees =
(8 1)!
2!

= 2520
or ( ) ( )
6
2
6 1 ! 7 1 ! C + = 2520
8i Solving both equations: 18.5 0.1 y x = + and 16.6 0.4 x y = +
25, 21 x y = =
5 25 125, 5 21 105 x y = = = =


8ii
2 2
125 105
2634
5
or 0.4 0.1 0.2
125 105
3215 2227.5
5 5
r

= =
| || |

| |
| |
\ .\ .

8iii
Let x = 26.
y =18.5+0.1 (26) = 21.1 ~ 21
The above equation is used because x is given and y has to be found.
The estimate is not reliable because r is small, i.e. it is close to 0, hence the linear correlation between x
and y is very weak.
9i
P (player wins the grand prize) = P (WWW)
=
2 4 6
8 10 12
| || || |
| | |
\ .\ .\ .
=
1
20

9ii
P (player wins a consolation prize)
= P (BWW or WBW or WWB)
=
6 2 4 2 6 4 2 4 6
8 10 12 8 10 12 8 10 12
| || || | | || || | | || || |
+ +
| | | | | | | | |
\ .\ .\ . \ .\ .\ . \ .\ .\ .

=
3
20

9iii
P (player wins a consolation prize/first draw was white) =
P (player wins a consolation prize and 1st draw was white)
P (1st draw was white)

=
P (WWB or WBW)
P (W)
=
1
10
1
4
=
2
5


P (player wins at least 2 grand prizes in his 4 attempts)
= 1 P (player did not win any grand prize) P (player wins only 1 grand prize)
=
4 3
19 1 19
1 4
20 20 20
| | | || |

| | |
\ . \ .\ .
= 0.0140

Alternative:
Let random variable X be the number of grand prizes a player wins in his 4 attempts at the game. Then
1
~ 4,
20
X B
| |
|
\ .
.
P (player wins at least 2 grand prizes in his 4 attempts)
= P ( 2) X >
= 1 P ( 1) X s
= 1 0.98598 = 0.0140
10i
X ~ B(108,0.04)
Assume independence of customers not turning up.
P(X = 0) = 0.96
108
= 0.0122
10
ii
Since n > 50, by CLT, the sample mean
4.1472
(4.32, )
60
X N approx.
( )
P 4 0.112 X s =
10
iii
3. Since n is large, np = 4.32 <5, X ~ Po(4.32) approx.
X
1
+X
2
+X
3
++X
m
= the number of customers that do not turn up for m consecutive days
4. Y = X
1
+X
2
+X
3
++X
m
~Po(4.32m)
5. P(Y 6) > 0.999
6. P(Y 5) < 0.001
7. Using G.C, m > 3.8
Or when m = 3, P = 0.011 > 0.001
8. when m = 4, P = 0.00055 < 0.001
Least number of days = 4
11
a
P(X >500) s0.01
Using GC, graphically 453 (3 s.f.) s
Alternatively,
P(X >500) s0.01
500
0.01
20
P Z
| |
> s
|
\ .

500
2.326 453 (3 s.f.)
20

> s
11
bi
Let X denote the amount of soft drink that is dispensed into a cup of 500 ml. X~N(470,20
2
)
P(X >500) = 0.0668 (3s.f.)
bii
Required Probability = (10.066807)
2
= 0.871
b
iii
X
1
+X
2
~N(940,800)
( )
1 2
1000 0.983 (3 s.f.) P X X + < =
biv
Let Y denote the number of cups that overflowed out of 300 cups.
Y ~ B(300,0.066807), np = 20.04217 > 5, nq = 279.95783 > 5,
Y ~ N(20.04217, 18.70321) (approximately)
P(Y > 20)
. c c
P(Y > 20.5) = 0.458 (3 s.f.)


2008 MJC H2 MATHS (9740) JC 2 PRELIM EXAM P1 SOLUTIONS


Qn Solution
1 Functions


(i)














(ii)




















( ) ( )
g f
, , 0, R D = =
For fg to exist,
g f
R D _ , largest domain of g is ( )
g
1, D = .



( ) ( ) | |
| |
g g fg
fg
1, 0, 1,1
1,1
f
D R D R
R
= = = =
=


















Qn Solution
2 Complex Number 1

Qn Solution
3 Integration Techniques




O
x
y

1






( )
( )
( ) ( )
( )
( )
2
3 3
2
3
2
2
3 3 3
2
3 3 3
2
ln d
1
ln 2ln d
3 3
2
ln ln d
3 3
2 1
ln ln d
3 3 3 3
2 2
ln ln
3 9 27
x x x
x x
x x x
x
x
x x x x
x x x
x x x
x
x x x
x x C
| |
=
|
\ .
=
( | |
=
( |
\ .
= + +
}
}
}
}


( ) ( )
2
2 2
2 1
1
d
1 2
1 4 1
d d
4 1 2 1 2
1 1
ln 1 2 tan 2
4 2
x
x
x
x
x x
x x
x x C

+
+
= +
+ +
= + + +
}
} }




Qn Solution
4 Mathematical Induction

Let
n
P be the statement
2
1
1
(2 1) (2 1)(2 1)
3
n
r
r n n n
=
= +

for n
+
e
For 1, n =
LHS =
2
1 1 =
RHS =
1
(2 1)(2 1) 1
3
+ =
i.e LHS = RHS
1
is true. P

Assume that
k
P is true for some k
+
e , i.e
2
1
1
(2 1) (2 1)(2 1)
3
k
r
r k k k
=
= +

.
Need to show
1 k
P
+
is true, i.e
1
2
1
1
(2 1) ( 1)(2 1)(2 3)
3
k
r
r k k k
+
=
= + + +

.
For 1, n k = +
LHS =
( )
1
2
2 2
1 1
(2 1) (2 1) 2( 1) 1
k k
r r
r r k
+
= =
= + +


( )
2 1
(2 1)(2 1) 2 1
3
k k k k = + + +

2
1
(2 1) 2 3(2 1)
3
k k k k ( = + + +



2
1
(2 1) 2 5 3
3
k k k ( = + + +



1
(2 1)(2 3)( 1)
3
k k k = + + + =RHS
Thus
1
is true is also true.
k k
P P
+

Therefore since
1 1
is true and is true is also true,
k k
P P P
+
then by mathematical
induction,
n
P is true . n
+
e






Qn Solution
5 Curve Sketching & Transformation
(i)
























(ii)






y

0


y
1
x












Qn Solution
6 Vectors 1 & 2

(a)



























1 1
2 2
3 3
,
a b
a b
a b
| | | |
| |
= =
| |
| |
\ . \ .
a b
1 1 2 2 3 3
1 1 2 2 3 3
| || | cos
a b a b a b
a b a b a b a b u
= + +
= + +
a b


2 2 2 2 2 2
1 2 3 1 2 3 1 1 2 2 3 3
2 2 2 2 2 2 2 2
1 2 3 1 2 3 1 1 2 2 3 3
cos
( )( ) cos ( )
a a a b b b a b a b a b
a a a b b b a b a b a b
u
u
+ + + + = + +
+ + + + = + +

2
2 1 1 2 2 3 3
2 2 2 2 2 2
1 2 3 1 2 3
2 2 2 2 2 2 2
1 1 2 2 3 3 1 2 3 1 2 3
( )
cos 1
( )( )
( ) ( )( )
a b a b a b
a a a b b b
a b a b a b a a a b b b
u
+ +
= s
+ + + +
+ + s + + + +


Alternative Method
| || | cosu = a b a b
| || | cos 1 u s s a b a b
2
2 2
| | | | s a b a b
If
1 1
2 2
3 3
,
a b
a b
a b
| | | |
| |
= =
| |
| |
\ . \ .
a b ,
2 2 2 2 2 2 2
1 1 2 2 3 3 1 2 3 1 2 3
( ) ( )( ) a b a b a b a a a b b b + + s + + + +




x
0


(b)


p 3q = 2p r
p 3q 2p r = 0
p (3q 2r) = 0
p // 3q 2r
p = k(3q 2r), k e










Qn Solution
7 Complex Numbers 2
















Qn Solution
8(a) Recurrence relations
(i)

(ii)
















(iii)
1
0.9 90, 1, 2, 3,...
n n
x x n

= + =

1
0.9 90
n n
x x

= +

( )
( )
2
2
2
2
3
3 2
3
1 2
0
0
0
0
0.9 0.9 90 90
0.9 0.9(90) 90
0.9 0.9 90 0.9(90) 90
0.9 0.9 (90) 0.9(90) 90
...
0.9 0.9 (90) ... 0.9 (90) 0.9(90) 90
1 0.9
0.9 90
1 0.9
0.9 900(1 0.9 )
0.9 ( 900) 90
n
n
n
n
n n
n
n
n n
n
x
x
x
x
x
x
x
x

= + +
= + +
= + + +
= + + +
=
= + + + + +
| |
= +
|

\ .
= +
= + 0(shown)


As , 0.9 0. 900
n
n
n x
(b) Let the sequences tend to o .
Thus 0.3 7 10 o o o = + =
Also
100
a o
o
= +
10 (10)
90
a
a
= +
=



















Qn Solution
9 Complex Numbers 3
























Qn Solution
10 Integration Applications
(a)






1
0
1
2
0
1
2
0
1
area of d (1)(1)
2
2 1
d
2
1
1
ln 1
2
1
ln 2
2
R y x
x
x
x
x
=
=
+
(
= +

=
}
}

(b)
( )
( )
1
2 2
0
2
1
2
0
2
1
2
0
2
2
2
4
2
0
2
2
4
0
4
0
4
0
4
0
2
1
volume of solid d (1) (1)
3
2
d
3
1
4
d
3
1
4tan
sec d
3
sec
4 sin d
3
1-cos2
4 d
2 3
2 (1-cos2 ) d
3
sin 2
2
2 3
4
2 3
y x
x
x
x
x
x
x
t
t
t
t
t
t t
t
t
t
t
u t
t u u
u
t
t u u
u t
t u
t
t u u
u t
t u
t t
=
| |
=
|
+ \ .
=
+
=
=
=
=
(
=
(

=
}
}
}
}
}
}
}











Qn Solution
11 Maclaurins Expansion
(a)






2 3
e 1 ...
2! 3!
x
x x
x = + + + +
Let x = 1,
2 3
1
1
1
1 1
e 1 1 ...
2! 3!
1
e 1
!
1
e 1
!
r
r
r
r

=
= + + + +
= +
=


(b)(i)
1
tan 2 y x

=
tan 2 y x =
Diff wrt x,
( )
2 2
d d
sec 2 1 4 2
d d
y y
y x
x x
| |
= + =
|
\ .

Diff wrt x,
( )
2
2
2
d d
1 4 8 0
d d
y y
x x
x x
| |
| |
+ + =
| |
\ .
\ .

Diff wrt x,
( )
3 2
2
3 2
d d d
1 4 16 8 0
d d d
y y y
x x
x x x
| | | |
| |
+ + + =
| | |
\ .
\ . \ .

When x = 0,
0, y =
d
2
d
y
x
= ,
2
2
d
0
d
y
x
= ,
3
3
d
16
d
y
x
=
3
8
2
3
y x x ~

(b)(ii)
1 1
1 3
5 5
0 0
1
5
2 4
0
8
tan 2 d 2 d
3
2
3
73
1875
x x x x x
x x

| |
~
|
\ .
(
=
(

=
} }


Alternative Method

Using GC, and converting answer to fraction



Qn Solution
12 Vectors 3
(i)






Method 1
Vector equation (1) in scalar product form is
0
3 2
1
r
| |
|
=
|
|
\ .
(3)
Vector equation (2) in scalar product form is
1
1 9
2
r
| |
|
=
|
|

\ .
(4)
1 2
i j k 5 5
0 3 1 1 1
1 1 2 3 3
n n
| | | |
| |
= = =
| |
| |

\ . \ .

a direction vector for line l is
5
1
3
m
| |
|
=
|
|
\ .
.
Hence equation of line l is
3 5
2 1 ,
4 3
r
| | | |
| |
= + e
| |
| |

\ . \ .
.

Method 2

3 2 y z + = (1)
1
1 9
2
| |
|
=
|
|

\ .
r in cartesian form is 2 9 x y z = (2)

Using GC Simult Eqn Solver:

Equation of line l is
29
3
5
2
1 ,
3
3
0
r
| |
|
| | |
|
|
= + e
|
|
|
|
\ .
|
|
\ .
.

(ii) A line through B perpendicular to plane
2
[ is

2 1
3 1 ,
1 2
r o o
| | | |
| |
= + e
| |
| |

\ . \ .
. (5)

Subst (5) into (4):

2 1
3 1 9
1 2 2
o
o
o
+ | | | |
| |
=
| |
| |

\ . \ .

2 3 2 4 9 o o o + + + + =

4
3
o =
Hence position vector OD

of foot of perpendicular from B to


2
[ is

4
2
3
10
4 1
3 5
3 3
11
8
1
3
OD

| |
+
|
| | |
|
|
= =
|
|
|
|
\ .
|

|
\ .


Now
10 3 1
1 1
5 2 1
3 3
11 4 1
AD
| | | | | |
| | |
= =
| | |
| | |

\ . \ . \ .



Hence length of projection of AB

onto
2
[ is

2 2 2
1 3
1 ( 1) 1
3 3
AD = + + =

.

(iii)
i j k 6 4 2
3 2 4 (9 20) 29
5 1 3 3 10 13
a m
| | | |
| |
= = + =
| |
| |

\ . \ .

vector equation of plane containing the line l and the origin is

2
29 0
13
r
| |
|
=
|
|

\ .
or
2
29 0
13
r
| |
|
=
|
|
\ .









Qn Solution
13 Differentiation and Applications
(a)

( )
( ) ( )
( )
1
2
2
1 2
2
2
2
2
2
2
2
1
1 4 8
d
2
cos 1 4
d
1 1 4
4
1 4
4
4
2 1 4
2
1 4
2
, 0
1 4
2
, 0
1 4
x x
x
x
x
x
x
x
x
x x
x
x x
x
x
x
x

=
=

>

<


(b)







r

Length of circle = 2 r t
Perimeter of hexagon = 2 d r t

total area A =
2
2
1 2
6 sin
2 6 3
d r
r
t t
t
| || | | |
= +
| | |
\ .\ . \ .

( )
2
2
3
2
24
r d r t t = +
( )( )
d 3
2 2 2
d 12
A
r d r
r
t t t = +
( )
d 3
2 2
d 6
A
r d r
r
t
t t =
( )
d 3
0 2 2 0
d 6
A
r d r
r
t = =
3 3
2
3 6
d
r
t
| |
+ =
|
|
\ .

0.075692 r d =
0.076 r d ~

2 2
2
d 3
2 0
d 3
A
r
t
t = + >
Thus, the sum of the areas enclosed by the two shapes is a minimum when the radius
of the circle is approximately 0.076d units.




2008 MJC H2 MATHS (9740) JC2 PRELIMINARY EXAM P2- SOLUTION

Qn Solution
1 Inequalities














( )
( ) ( )
( ) ( )
2
2
2
2
2
2
2
2
2
6
1
4 5
6 4 5
0
4 5
6 4 5
0
4 5
4 1
0
4 5
4 1
0
4 5 1
Since 4 1 0 for ,
4 5 1 0
5
1
4
x
x x
x x x
x x
x x x
x x
x
x x
x
x x
x x
x x
x

>
+
+
>
+
+
>
+

>
+
+
s
+
+ > e
+ <
< <


Qn Solution
2 AP & GP































For series to converge,
2
3 2
1
1
a a
a
+
<



( 1)( 2)
1
1
a a
a

<


1 2 1 a < <
1 3 a < <

Also, ( 1)( 2) 0 a a > ( all terms in GP positive)
1 or 2 a a < >

{ } set of values of : 2 3 a a a = e < <

Alternative Solution:

For sum to infinity to exist and all terms positive,

0 1
0 2 1
2 3
r
a
a
< <
< <
< <



Given
11
4
a = .
11 3
2
4 4
r = =

7 3
1
4 4
6.999
3
1
4
n
(
| |

(
|
\ .
(

>



3 6.999
1
4 7
n
| |
>
|
\ .


3 6.999
1
4 7
n
| |
<
|
\ .


4
ln(1.4286 10 )
3
ln
4
n

>
30.776 n >

least 31 n =


Qn Solution
3 Summation







( ) ( ) ( )( )
1 1 1 ( 1)( 2) 2 ( 2) ( 1)
2 1 2 2 2 1 2
x x x x x x
x x x x x x
+
+ =


=
( )( )
2 2
3 2 3
2 1 2
x x x x
x x x
+ +


=
( )( )
2
2 1 2 x x x


( )( )
1
1 2 x x x
=

(verified)


( )( ) ( ) ( )
3 3
1 1 1 2 1
1 2 2 1 2
1 2 1

3 2 1
1 2 1

4 3 2
1 2 1
1
5 4 3
2
1 2 1
1 2 3
1 2 1

1 2
N N
n n
n n n n n n
N N N
N N N
= =
(
= +
(


(
+
(
(
(
+ +
(
(
(
+ +
= (
(
(
(
+ +
(

(
(
+ +
(




( )
1 1 1 1 2
2 2 1 1
1 1 1 1
2 2 1
1 1

4 2 1
N N N
N N
N N
| |
= + +
|

\ .
| |
= +
|

\ .
=



Note that
( )( ) ( ) ( )
3
1 1 1 1 1
1 2 2 1 2 2 n n n n n n n
(
s = +
(




( )
3 3 3
1 3
3
1
1 1 1 1
1 2
1 1 1
< 1 .
8 4 2 1
1 1 1 11
1 .
8 4 8
N N
n n
n
n n
N N
n
= =

=
= + +
+ +

< + + =














Qn Solution
4 Binomial Expansions

(a)



8 2 3
2 3
(8)(7) (8)(7)(6)
(1 ) 1 8 ...
2! 3!
1 8 28 56 ...
x x x x
x x x
+ = + + + +
= + + + +


let
2
x y ky = +

2 8 2 2 2 2 3
(1 ) 1 8( ) 28( ) 56( ) ... y ky y ky y ky y ky + + = + + + + + + +
From
2 2
28( ) y ky + , term in
3 3 3
: 28(2 ) 56 y ky ky =
From
2 3
56( ) y ky + , term in
3 3
: 56 y y
Given the coefficient of
3
y is zero, 56 56 0 1 k k + = =

(b)
1 1
1
1
2
2
2
2
2
2 3 1 1
( 1)( 2) 1 2
( 1) ( 2)
1
(1 ) 1
2 2
( 1)( 2) 1 ( 1)( 2)
1 ( 1) ... 1 ( 1) ...
2! 2 2 2! 2
1
1 ...
2 4 8
3 5 9
...
2 4 8
x
x x x x
x x
x
x
x x
x x
x x
x x
x x

+
= +
+ + + +
= + + +
| |
= + + +
|
\ .
(
( | | | |
= + + + + + + + (
| |
(
\ . \ . (

= + + + +
= + +

















Alternative Solution:
1 1
1
1
2
2
2
2
2 3
(2 3)( 1) ( 2)
( 1)( 2)
1
(2 3)(1 ) 1
2 2
1 ( 1)( 2) ( 1)( 2)
(2 3) 1 ( 1) ... 1 ( 1) ...
2 2! 2 2! 2
1
(2 3)(1 ...)(1 ...)
2 2 4
1
(2 3)
2
x
x x x
x x
x
x x
x x
x x x
x x
x x x
x

+
= + + +
+ +
(
| |
= + + + (
|
\ . (

(
( | | | |
= + + + + + + + (
| |
(
\ . \ . (

= + + + + +
= +
2
2
3 7
(1 ...)
2 4
3 5 9
...
2 4 8
x x
x x
+ +
= + +




Qn Solution
5 Differential Equations
(a)






d
d
y
z x
x
=
2
2
d d d
d d d
z y y
x
x x x
= + #
Given
2
2
2
d d
3
d d
y y
x x
x x
+ =
2
d
3
d
z
x
x
=

Hence
2
d 3 d z x x =
} }


3
, z x c c = + e

d
d
y
x
x
=
3
x c +
2
d
d
y c
x
x x
= +

3
ln , ,
3
x
y c x d c d = + + e






(b)

d
(1000 )
d
k
t
u
u =
1
d d
1000
k t u
u
=

} }

ln(1000 u) = kt + C
1000
1000 where
kt C
kt C
e
Ae A e
u
u


=
= =

when t = 0, u = 40
40 = 1000 A A = 960
when t = 1, u = 160
160 1000 960
840 7
960 8
7
1000 960
8
k
k
t
e
e
u

=
= =
| |
=
|
\ .





Qn Solution
6 Permutation and Combination



(a)


(b)



No of ways =
11 8 5 2
3 3 3 2
15400
3!
| || || || |
| | | |
\ .\ .\ .\ .
=

No of ways = ( ) ( )( ) 8 1 ! 4 3 2 =120960











Qn Solution
7 Sampling Method

(i)

Simple random sampling is
- free from bias
- relatively small sample size, hence easy to conduct
- analysis of data relatively easy

Disadvantage: May not be easy to get access to members who have been chosen for the
sample

(ii)
Systematic sampling
- Simple to use for large number of walk-in customers
- Sample obtained more evenly spread over the spectrum of walk-in customers
- Relatively easy to conduct

The hobby shop could decide on an interval to select a walk-in customer for the survey, for
example, every 20
th
customer. It then chooses a random start between the numbers 1 and 20.
This will determine the first customer to be surveyed and every 20
th
customer thereafter will
be surveyed.



Qn Solution
8 Binomial Distribution
(i)


Let X be the number of students out of n who use AIKON brand mobile phones.
When n = 30,
1
~ 30,
5
X B
| |
|
\ .

Expected number of students who use AIKON brand phone
=
( )
1
30 6
5
| |
=
|
\ .

P( ) 5 0.172 X = =

(ii)
( )
( )
( )
( )
P 1 0.99
1 P 0 0.99
P 0 0.01
4
0.01
5
lg 0.01
20.6
4
lg
5
n
X
X
X
n
> >
= >
= <
| |
<
|
\ .
> =
| |
|
\ .

Hence, least n = 21.

Qn Solution
9 Estimation and Hypothesis Testing
(i)




Unbiased estimate for population mean = 1.96
20
x
x = =




Unbiased estimate for population variance =
( ) ( )
2
2
2 2
39.2
1 1
77.022
1 19 20
x
s x
n n
| |
| |
|
= = |
|
|
\ .
\ .


= 0.01

(ii)
0
1
H : 2
H : 2

=
<

Assumption:
( )
2
~ , X N o
Test statistic:
X
T
s
n

=
Level of significance: % o
Critical region: Reject
0
H if p-value < %
100
o

Assuming
0
H is true, from the GC, p-value = 0.044797
= 0.0488 (3 s.f)
Set of values of { } : 4.48 100 o o o = e s s

Qn Solution
10 Probability
(i)




AB represents the event the card taken is red and numbered 1.

P(AB) =
1
40
.

Note: If students use the formula that ( ) ( ) ( ) P = P P , A B A B they need to prove that
event A and event B are independent.








(ii) AB represents the event the card taken is either red or numbered 1.

P(AB) = P(A) + P(B) P(AB)
=
10 4 1
40 40 40
+
=
13
40


(iii)
( ) ' '
P( ' | ')
( ')
P A B
A B
P B

=
=
13
1
40
36
40


=
3
4


Required probability =
4
1 1 1
3! 10
3 40 39 38
| |
| || || |

| | | |
\ .\ .\ .
\ .

= 0.00405


Qn Solution
11 Poission


(i)
Let X be the no of requests for the hire of a truck on a day.
Po(2) X






(ii)



Prob at most 1 truck not hired out
P( 2)
1 P( 1)
0.59399
0.594(3sf)
X
X
= >
= s
=
=


Prob that requests can be met on a given day
( 3)
0.85712
0.857(3sf)
P X = s
=
=






Prob that a particular truck is not used
2 2 1
P( 0) P( 1) P( 2)
3 3 2
0.40601
0.406 (3sf)
X X X = = + = + =
=
=






Let the no of trucks be n
P( ) 0.01 X n > <
P( ) 0.99 X n s >
From GC,
P( 5) 0.98344 0.99
P( 6) 0.99547 0.99
X
X
s = <
s = >

least no of trucks is 6


Qn Solution
12 Normal Distribution

Let F = the length, in cm, of a floor tile.
( )
2
~ N 30, 0.2 F
Let L = the length, in cm, of a wall tile.
( )
2
~ N 10, 0.15 L

(i)
( )
1 2 3 4 5
~ N 150, 0.2 F F F F F + + + +
( )
1 2 3 4 5
P 150.5 151.5 F F F F F < + + + + <
0.13138 =
0.131 = (3 s.f.)

(ii)
( )
1 2 3 4 5 6
~ N 60, 0.135 S L L L L L L = + + + + +
( )
2
2 ~ N 60, 0.4 W F =
( ) P 1 S W < +
( ) P 1 S W = < ( ) ~ N 0, 0.295 S W
0.96720 =
0.967 =
(iii)
( )
2 1 2 25
...
~ N 30, 0.04
25
F F F
F
+ + +
=
( )
P 30 0.02 F a > =
P 0.02
0.04
a
Z
| |
> =
|
\ .

P P 0.02
0.04 0.04
a a
Z Z
| | | |
s + > =
| |
\ . \ .

P 0.01
0.04
a
Z
| |
s =
|
\ .

2.32635
0.04
a
= . 0.0931 a = (3 s.f.)




Qn Solution
13 Regression and Correlation

(i)

Suitable to use on h s since s is clearly, the controlled [independent] variable.


(ii)

Let 20 x s = and 100 y h =

Regression line of on y x is:
( )
y y b x x = where
1 143 1 391
9.5333, =26.067
15 15
x x y y
n n
= = = = =


( )
2 2
2
143 391
484
15
and 3.0899
143
2413
15
x y
xy
n
b
x
x
n


= = =
| |
|
\ .


( ) 26.067 3.0899 9.5333 55.523 3.0899 y x y x = =
Thus the line of on h s is:
( ) 100 55.523 3.0899 20 217.32 3.0899 217 3.09 [3 sf] h s h s h s = = =


(iii)


(iv)





(v)



For every extra revolution/minute the life of the drill reduces by about 3 hours.


Find the product moment correlation coefficient, r and if 1 r ~ then the calculated regression
line may be a good fit to the data.
OR
If the raw data is available then a scatter diagram could be drawn with the regression line on it.
If the points lie close to the regression line, then the line is said to fit the data well.

At
217.32 125
125, 3.0899 217.32 125 =29.9 30 revs/minute
3.0899
h s s

= + = = ~





RJC 2008 Preliminary Exam Paper 1 Solutions

Q1
[6]



























Let
n
P be the statement
1
2
(3 2 )3 3
9
( 1)
n
r n
r
r
r r n
+
=

, , 2 n n
+
e > .

When 2 n = , LHS
2
(3 4)3 9
(2)(1) 2

= =
RHS
3
3 9
9
2 2
= = = LHS.
Since LHS = RHS,
2
P is true.

Assume that
k
P is true for some , 2 k k
+
e > , i.e.
1
2
(3 2 )3 3
9
( 1)
k
r k
r
r
r r k
+
=

.
To show
1 k
P
+
is true, i.e.
1
2
2
(3 2 )3 3
9
( 1) 1
k
r k
r
r
r r k
+
+
=

=
+

.

LHS =
( )
1
1
2 2
3 2( 1) 3
(3 2 )3 (3 2 )3
( 1) ( 1) ( 1)( 1 1)
k k
k
r r
r r
k
r r
r r r r k k
+
+
= =
+

= +
+ +


1 1
3 (1 2 )3
9
( 1)( )
k k
k
k k k
+ +

= +
+


1
3 [ 1 (1 2 )]
9
( 1)
k
k k
k k
+
+
=
+

1
3 (3 )
9
( 1)
k
k
k k
+
=
+


2
3
9
1
k
k
+
=
+
= RHS

1 k
P
+
is true whenever
k
P

is true.
Since
2
P is true, by mathematical induction,
n
P is true for all , 2 n n
+
e > .















Q2
[3]







Let
2
2
( 1) 1 1
A B C
r r r r r
= + +
+


2 ( 1)( 1) ( 1) ( 1) A r r Br r Cr r = + + + +
0, 2
1, 1
1, 1
r A
r B
r C
= =
= =
= =






[4]



























2
2 2 1 1
( 1) 1 1 r r r r r
= + +
+


2
1 1 1 1
...
(4)(15) (5)(24) (6)(35) ( )( 1)
N
S
N N
= + + + +

2
4
1
( 1)
N
r
r r
=
=


4
1 1 2 1
2 1 1
N
r
r r r
=
| |
= +
|
+
\ .


1 1 2 1
[
2 3 4 5
= +

1 2 1
4 5 6
+ +

1 2 1
5 6 7
+ +

1 2 1
6 7 8
+ +
+

1 2 1
2 1 N N N
+ +



1 2 1
]
1 1 N N N
+ +
+


1 1 1 1
[ ]
2 12 1 N N
= +
+
1 1
24 2 ( 1) N N
=
+


As N ,
1
0
2 ( 1) N N

+
, Limit of
1
24
N
S =
Q3
[1]


[4]






[2]
(i) range of f
1
= domain of f = (, 3), domain of g = .
Since range of f
1
_ domain of g, the function gf
1
exists.

(ii) Let y = 3 x. Then x = 3 y.
f
1
: x 3 x, x > 0.
gf
1
(x) = g(3 x) = 2(3 x)
2

Domain of gf
1
= domain of f
1
= (0, )
Range of gf
1
= [0, )

(iii)
1
2
gf
1
(x) =
1
2
g(3 + x) = (x + 3)
2
Translate the graph of y = g(x) for 0<x<3 3 units in the negative x direction, followed by a
stretch parallel to the y-axis with a scale factor of
1
2



Q4

[2]



[3]




The mass of sand spread by the gardener follows an AP with first term 5 and common difference 2.
(i) Mass of sand he has spread by the end of the n th day,
[2(5) ( 1)(2)] (8 2 ) (4 )
2 2
n
n n
S n n n n = + = + = + .

(ii) We are finding the least integer n such that
(4 ) 1500
n
S n n = + >

2
4 1500 0 n n + >
40.8 n s (rejected since n
+
e ) or 36.8 n >










[2]





[1]
Alt :
From the GC, when

2
2
36, 4 1500 60
37, 4 1500 17
n n n
n n n
= + =
= + =

Hence minimum number of days required for him to spread the 1500 kg of sand is 37.

The mass of sand spread by the gardeners neighbour follows a GP with first term 75 and common
ratio 0.95.

(iii) Mass of sand he has spread by the end of the n th day,

75(1 0.95 )
1500(1 0.95 )
1 0.95
n
n
n
S

= =

.

(iv) Since
75
1500
1 0.95
S

= =

, which is the mass of sand to be spread, the neighbour will never


complete his task with the approach taken.

Q5
[1]


[4]











[3]
The differential equation relating P and t is

dP
kP
dt
= , where k is a positive constant.
Solving this equation, we have
1
dP kdt
P
=
} }

ln P kt C = +
When 0 t = , 32000 P = ,
ln(32000) (0) k C = +
ln(32000) C = .
So ln ln(32000) P kt = +

ln32000
e .e
kt
P

=
32000e
kt
P

= (Shown)

In 2006, i.e. when 10 t = , 2000 P = ,
ln(2000) 10 ln(32000) k = + or
10
2000
e
32000
k
=
1 2
ln(16) ln 2
10 5
k = =
So the equation relating P and t is
2ln2
5
32000
t
P e
| |

|
\ .
=
In 2007, i.e., when 11 t = , the value of the car has depreciated to
( )
2ln 2
5
(11)
32000 1516 800 e

~ > .
Thus the man sold his car below the price predicted by the model.

Q6
[1]

[4]










(i)
1
( 2 )
3
OC = + a b using ratio theorem
(ii) Length of projection of OC onto OB





[3]









2
.
1
. .
3
2
1
1 2(2)(2) since .
6
3
2
OC
=
+
=
= + =
=
b
b
a b 2bb
bb b


3 3
2 4
ON = =
b
b
b

(iii)

3 1 1 1
( 2 )
4 3 3 12
3 3
( ) ( )
4 4
CN
ND
= + = +
= + = +
b a b a b
a b b a b

Since the points C, N and D are collinear,

for some
3 1 1
i.e. ( ) ( )
4 3 12
ND CN o o
o
= e
+ = + a b a b

Since is not parallel to and and non-zero a b a b ,

3
1 and =
3 4 12
1
= 3 and =
2
o o

o
=


Alt :
. 0
3 3
. [( ) ]. [ ( ) ]. 0
4 4
3
1 4( ) 0
4
1
Thus = .
2
ND
ND

=
= + = + =
+ =
b
b a b b b a b b
















Q7
[2]




(i)













[2]

[5]








(ii) Area of
( )
3
4
1
2 1 2 d 3.50 R x x = =
}
(3 s.f.).
(iii) The two curves intersect at (1, 0) and (2,1).
Required volume ( ) ( )
2 2
4 4
1 1
1 2 d 1 d x x x x t t =
} }


2
5 5
1
( 2) ( 1)
5 5
3
5
x x
x t
t
(
=
(

=

Q8











[2]


















[3]





1
8 1
: 12 7 ,
16 2
l s s
| | | |
| |
= + e
| |
| |

\ . \ .
r
2
4 2
: 6 1 ,
8 2
l t t
| | | |
| |
= + e
| |
| |

\ . \ .
r
(i) As PQ is perpendicular to
1
l and
2
l , it is perpendicular to
1
7
2
| |
|
|
|

\ .
and
2
1
2
| |
|

|
|
\ .

and parallel to
1 2 12 4
7 1 6 3 2
2 2 15 5
| | | | | | | |
| | | |
= =
| | | |
| | | |

\ . \ . \ . \ .

i.e. PQis parallel to the vector 4 2 5 i j k . (shown)







(ii) As Q lies on
2
l ,
4 2
6 1
8 2
OQ t
| | | |
| |
= +
| |
| |

\ . \ .
for some t e.


































































4 2 8
6 1 12
8 2 16
PQ t
( | | | | | |
( | | |
= +
( | | |
| | |
(

\ . \ . \ .
12 2
6 1
24 2
t
| | | |
| |
= +
| |
| |

\ . \ .

*Since PQis parallel to 4 2 5 i j k ,

12 2 4
6 1 2
24 2 5
PQ t o
| | | | | |
| | |
= + =
| | |
| | |

\ . \ . \ .

Re-arranging,
2 4 12
1 2 6
2 5 24
t o
| | | | | |
| | |
=
| | |
| | |

\ . \ . \ .

Solving using G.C, 2 t = .

8
4
4
OQ
| |
|
=
|
|

\ .

*OR
As PQ is perpendicular to
1
l ,
1
7 0
2
PQ
| |
|
=
|
|

\ .


12 2 1
6 1 . 7 0
24 2 2
t
( | | | | | |
( | | |
+ =
( | | |
| | |
(

\ . \ . \ .


12 42 48 (2 7 4) 0
8
2 and 4
4
t
t OQ
+ + =
| |
|
= =
|
|

\ .

*OR
As PQ is perpendicular to
2
l ,
2
. 1 0
2
PQ
| |
|
=
|
|
\ .


12 2 2
6 1 . 1 0
24 2 2
t
( | | | | | |
( | | |
+ =
( | | |
| | |
(

\ . \ . \ .


8
24 6 48 9 0. 2 and 4
4
s t OQ
| |
|
+ + = = =
|
|

\ .

Alt:

Line PQ :
8 4
12 2 ,
16 5
u u
| | | |
| |
= + e
| |
| |

\ . \ .
r




























[2]





[3]




2
4 2
: 6 1 ,
8 2
l t t
| | | |
| |
= + e
| |
| |

\ . \ .
r
Q is the intersection of the 2 lines.


8 4 4 2
12 2 6
16 5 8 2
u t
u t
u t
+ = +
=
= +


i.e.
4 2 12
2 6
5 2 24
u t
u t
u t
=
+ =
=

Using G.C, 2 t = and
8
4
4
OQ
| |
|
=
|
|

\ .
.


(iii) Equation of
4 8 4
: . 2 12 . 2 136
5 16 5
| | | | | |
| | |
H = =
| | |
| | |

\ . \ . \ .
r
Therefore the cartesian equation is 4 2 5 136 x y z =

(iv)









Let the image of Q after reflection in H be Q.
Then
1
( ')
2
OP OQ OQ = +

8 8 24
' 2 2 12 4 20
16 4 36
OQ OP OQ
| | | | | |
| | |
= = =
| | |
| | |

\ . \ . \ .

As
3
l is // to
2
l , a vector equation of
3
l is
24 2
20 1 ,
36 2

| | | |
| |
= + e
| |
| |
\ . \ .
r

Q9


Complex Nos

Q10
[3]



(a) (i)
( )
2 1
2 2 2
1 1 2 1 1
d d d ln 1 tan
1 2 1 1 2
x x
x x x x x C
x x x

+
= + = + + +
+ + +
} } }



Q



P


Q


[4]












[3]













[3]

(ii) With ( 3) u x = ,
d 1 1
,
d 2 ( 3) 2
u
x x u
= =

so

2
4 2
5 3
5 3
2 2
3
2
(5 3) ( 3) d (5 12)( )(2 )d
(10 24 ) d
2 8
2( 3) 8( 3)
2( 1)( 3)
x x x u u u u
u u u
u u C
x x C
x x C
= +
= +
= + +
= + +
= + +
} }
}

(b)

( )
2 2 2
2
0 0
0
0
e sin 2 d e sin 2 e 2cos 2 d
=e sin 2( ) e sin 2(0) 2 2
2
x x x
I x x x x x
J J
t t t
t
t
( = =

=

} }
and

( )
2
2 2 2
2
2 2
0 0
0
0
0
1 2 2 1
e cos 2 d e cos 2 e 2sin 2 d
= e cos 2( ) e cos 2(0) 2 e sin 2 d
2
e e
x x x
x
I I
J x x x x x
x x
t
t t t
t
t t
t
+ =
( = =

+
=

} }
}

Hence
2 2
1
2( 2 ) e 1 e 1
5
J J J
t t
| |
= = +
|
\ .
and
2
2
e 1 .
5
I
t
| |
= +
|
\ .

Q11
[3]















[2]






[1]

(a)
ln
2
ln 2
d 1
d ln 2
d 1
When , [shown]
d ln 2
ln
1
ln 2
ln 2
1 1 ln
ln 2 ln 2 ln 2
y
x
x y
y
x x
y
x a
x a
a
y
x a a
a
y x
a

= =
=
= =
+
=

= +


1 ln
0 e
ln 2 ln 2
a
a = =
Equation of tangent passing through origin is
1
eln 2
y x =


From the graph, range of m is
1
0
eln 2
m < < .
Q11

[1]


















[4]














[2]





(b)













(i)
1
tan ABD
x
Z = and
6
tan ABC
x
Z =
1 1
6 1
tan tan ABC ABD
x x
u

| | | |
= Z Z =
| |
\ . \ .
[shown]
(ii)
2 2 2 2
d 1 6 1 1
d
6 1
1 1
x x x
x x
u | | | |
=
| |
\ . \ .
| | | |
+ +
| |
\ . \ .


2 2
6 1
36 1 x x

= +
+ +

For stationary value,
2 2
d 6 1
0 0
d 36 1 x x x
u
= + =
+ +


2 2
6( 1) 36 0 x x + + + =
2
5 30
6 or 6 (NA)
x
x
=
=

The required distance is 6 m.

(iii)

From the graph we obtain minimum value when x=15.
The required angle is 18 .




1
5
A
D
B
C
y A
RJC 2008 Preliminary Exam Paper 2 Solutions

1
[1]





[3]









[2]

2 2 2 2 2 2 2
( 2) 1 ( 2 ) 1 ( 2 1)( 2 1) x x x x x x x x + = + = + + +

2 2 2
( 1) ( 1) ( 2) x x
(
= + +



( )( )
2
( 1) 1 2 1 2 x x x = + + + + (shown)

2
2
1
, 2
( 2)
x x
x
> =
+

2 2
( 2) 1 x x + s since
2
( 2) 0 x + >
( )( )
2
( 1) 1 2 1 2 0
1 2 2 2 1 2
x x x
x x
+ + + + s
s < < s +

or



2
2
1
e
(e 2)
x
x
>
+

Replacing x by e
x
in the above, we get
1 2 2 2 1 2
x x
e e s < < s + or

Since e 0
x
> ,
( )
ln 1 2 x s +
Range of values of x is
( ) (
, ln 1 2
(
+

.

2
[2]













[4]






(i) To produce $1 of Y, $0.25 of X is needed.
To produce $y of Y, $0.25y of X is needed.
Similarly, to produce $x of X and $z of Z, $0.1x and $0.2z of X are needed respectively.

For product X,
Worth of internal provision $(0.1 0.25 0.2 ) x y z = + +
Worth of production $x =
Worth of external demand = $ 50000.

Since internal provision + external demand = production, we have
0.1 0.25 0.2 50000 x y z x + + + =
0.9 0.25 0.2 50000 x y z = --- (1)

(ii) Similarly, for products Y and Z, we have
0.3 0.6 0.5 75000 x y z + = --- (2)
and 0.1 0.15 0.9 125000 x y z + = --- (3)

Solving (1), (2) and (3) using GC, we obtain
229921, 437795, 237402 x y z = = = (to nearest integer)
3
[2]
[2]
[2]
Complex No
4
[1]

[2]

[3]

(i) The graph of
1
f( )
y
x
= is decreasing for (4, ) xe .
(ii) The graph of f '( ) y x = is below the x -axis for ( , 2) (2, 4) xe .

(iii)


1
1 1+
2

\2
y
y = \2













[3]












(iv)















5(a)
[3]

2 2 2 2 2
( 1)
1 (2 3 ) 1 (2 3 ) (2 3 ) ... (2 3 )
2
n
n
n n
x x n x x x x x x

( + + = + + + + + + +


Coefficient of
2
( 1)
3 (4) 105
2
n n
x n

= + =

2
2 105 0 n n + =
(2 15)( 7) 0 n n + =

15

2
n = (Rejected since n
+
e ) or 7 n =
5(b)
[4]

1
2 2 4
2
1 4 1
(1 4 ) 3(1 ) (1 4 ) 1 ...
3 3 3 3 9
x x x x
x x
x

( | |
= + = + +
| (
+
\ .


2 3 4
1 4
1 4
3 3 3 9
x x x
x
| |
~ + +
|
\ .


The expansion is valid for
2
3 x < 3 3 x < <
( 3, 3) x e

5(c)
[3]











2
d
2 1
d
y
y y
x
= ----- (1)
Differentiate (1) w.r.t. x , we get
2
2
2
d d d
2 2 2
d d d
y y y
y y
x x x
| |
+ =
|
\ .


2
2
2
d d d
d d d
y y y
y y
x x x
| |
+ =
|
\ .
----- (2)
When 0 x = , 3 y = ,
d 4
d 3
y
x
=

2
4
0
(4,2)
y
x
(4,2)
2
x=2 x=2
y=4
2




[3]







Subst into (2),
2
2
2
d 4 4
3 3
d 3 3
y
x
| | | |
+ =
| |
\ . \ .

2
2
d 1 16 20
4
d 3 9 27
y
x
| |
= =
|
\ .
(shown)


Differentiate (2) w.r.t. x , we get
2
3 2 2 2
3 2 2 2
d d d d d d d
2
d d d d d d d
y y y y y y y
y y
x x x x x x x
| | | |
| | | | | |
+ + = +
| | | | |
\ . \ . \ .
\ . \ .


2
3 2 2
3 2 2
d d d d d
3
d d d d d
y y y y y
y y
x x x x x
| |
| | | |
+ = +
| | |
\ . \ .
\ .


When 0 x = , 3 y = ,
d 4
d 3
y
x
= ,
2
2
d 20
d 27
y
x
= ,
3
3
d 28
d 81
y
x
=

By Maclaurins Thm,

2 3
4 20 28
3 ...
3 27 2! 81 3!
x x
y x
| | | |
= + + + +
| |
\ . \ .


2 3
4 10 14
3 ...
3 27 243
x x x = + + + +

6
[1]


[3]


[2]
(i) Number of ways ( )
3
12 8 4
4 4 4
4! C C C = or 12! = 479001600

(ii) Number of ways
( )
( )
2
3
6 4 2
2 2 2
4! 111974400 C C C = =

(iii) Number of ways
( )
( )
2
3
6 4 2
2 2 2
2 2! 2! 4147200 C C C = =

7
[4]







[1]

[1]

(i) Method 1: Simple Random Sampling
Each staff in the company is assigned a number from 1 to 240. Then a list of 30 random
numbers chosen from 1 to 240 is used to select the sample of 30 staff.

Method 2: Stratified Sampling
The sample of 30 staff is made up of staff from all 6 departments. The staff are selected at
random (using simple random sampling or systematic sampling) from each department, with
the number selected being proportional to the number of staff in the department.

(ii) Method 1 is easier to implement as there is no need to consider the relative sizes of the
departments.
Method 2 ensures that all departments are fairly represented, Method 1 may not.

8
[3]






[3]
(i) Let X be the number of flaws in a roll of a particular design of wallpaper.
Then Po(0.15) X .
Required probability 2P( 1)P( 1) X X = = >
| |
2P( 1) 1 P( 1) X X = = s
0.00263 = (3 s.f.)

(ii) Since 50 n = is large, by Central Limit Theorem,
0.15
N 0.15,
50
X
| |
~
|
\ .
approximately.
ie. ( ) N 0.15, 0.003 X ~ approximately.
P( 0.3) 0.00309 X > = (3 s.f.)

9
[2]

(i) The t -test should be used because the sample size is small and the population variance is
unknown.

[5]




(ii) Let X be the distance thrown by the shot putter, in metres.

0
H : 8.92 = vs
1
H : 8.92 >

Perform a 1-tail test at 5% level of significance.

Assume that X follows a normal distribution.

Under
0
H ,
0
( 1)
/
X
T t n
S n

= , where
0
8.92, 0.099783, 8.983, 10 s x n = = = =

Using a t -test, p -value 0.0385 = (3 s.f.)
Since the p -value = 0.0385 < 0.05, we reject
0
H and conclude that there is sufficient
evidence, at 5% level of significance, that the shot putter has improved.

10

[1]







[3]







[4]





Given P(win)
1
2
= and P(defeat)
1
6
=
1
P(draw)
3
=
(i) P(1 draw and 1 defeat) = P(1 draw, 1 defeat, 2 wins)

2
1 1 1 4!
3 6 2 2!
| |
=
|
\ .


1
6
= (shown)

(ii) P(wins the first match and goes on to win exactly one other match)

1
2
= P(wins one other match and loses or draws in the other 2)

2
1 1 1 3! 3
2 2 2 2! 16
(
| |
= =
(
|
\ .
(



(iii) P(wins exactly one match | obtains four pts)


P(wins exactly one match and obtains four points)
P(obtains four points)
=


P(1 win, 1 draw, 2 defeats)
P(1 win, 1 draw, 2 defeats) P(4 draws)
=
+



2
2 4
1 1 1 4!
2 3 6 2!
1 1 1 4! 1
2 3 6 2! 3
| |

|
\ .
=
| | | |
+
| |
\ . \ .


1
9
18
1 1
11
18 81
= =
+


11
[3]





[5]

(i) Let X be the number of defective articles, out of 10.
Then B(10, 0.05) X .

P( 1) 0.31512 X = = (5 s.f.)
= 0.315 (3 s.f.) (shown)

(ii) Let Y be the number of samples, out of 80, that will each contain exactly one defective
article.







Then B(80, 0.315) Y .

Since 80 n = is large such that (80)(0.315) 25.2 5 np = = > and
(1 ) (80)(1 0.315) 54.8 5 n p = = > , ( ) N 25.2,17.262 Y ~ approximately

P(25 35) P(24.5 34.5) Y Y s < = < < (by continuity correction)
0.554 = (3 s.f.)
12
(a)



[4]





Given that the diameter of a randomly chosen lid is X cm and the diameter of the top of a
randomly chosen container into which the lid is pressed is Y cm,
where
2
N(12.1, 0.03 ) X ~ and
2
N(12.0, 0.03 ) Y ~ .

2 2 2
1 2 3
2 ~ N( 3 12.1 2 12.0, 3 0.03 2 0.03 ) X X X Y + + + + +
ie.
1 2 3
2 ~ N(60.3,0.0063) X X X Y + + +

1 2 3
P( 2 60.5) 0.994 X X X Y + + + < = (3 s.f.)

(b)
[3]




[2]
(i)
2
N(0.1, 2 0.03 ) X Y ~
ie. N(0.1, 0.0018) X Y ~

Probability that a pairing is accepted P(0.02 0.17) X Y = < < = 0.921 (3 s.f.)

(ii) Number of trials required to obtain 2000 accepted pairings

2000
2171.9 2172
0.92085
= ~ = (nearest integer) or 2170 (3 s.f.)

13
[1]
(i) Value of the product moment correlation coefficient is 0.807 (3 s.f.).

[3]

(ii)











The value found in (i) suggests there is some form of
negative linear correlation between x and y but the
scatter diagram shows that the relationship between
x and y is non-linear. Thus, the value of the
product moment correlation coefficient is not a good
indication of the linearity between x and y.

[2]






[1]


[3]

(iii) From the scatter diagram, we see that y decreases as x increases, which is the case for
model B ( , 0
b
y a b
x
= + > ).
For models A (
2
y a bx = + ) and C ( ln y a b x = + ), y increases as x increases, since 0 b > .
Hence model B is appropriate.

(iv) Value of the product moment correlation coefficient between y and w is 0.976 (3 s.f.).

(v) Equation of the regression line of y on w is 0.86652 3.2758 y w ~ + (5 s.f.),
i.e. 0.867 3.28 y w = + (3 s.f.).

When 1.4 y = , 1.4 0.86652 3.2758 0.16285 6.14 6 w w x ~ + ~ ~ ~ .
An estimate of the number of pens ordered when a unit price of $1.40 is quoted is 6.

This estimate is highly reliable since 1.4 y = is between the range of values of y from 0.80 to
1.50, and the product moment correlation coefficient between y and w is 0.976 (3 s.f.), which
suggests a strong positive linear correlation between y and w.
y
x


TJC 2008 Prelim Paper 1 Solutions
1 The lines l
1
and l
2
have equations
3
2
1
2 2
3
a

| | | |
| |
= +
| |
| |
\ . \ .
r and
4 2
0 4
1 b

| | | |
| |
= +
| |
| |

\ . \ .
r where , e and
, a be .

(i) Given that l
1
and l
2
are parallel, write down the values of a and b. [2]

(ii) Find the shortest distance between l
1
and l
2
. [3]

(iii) Find a vector equation of the plane containing l
1
and l
2
. [1]

Solution:
(i) a = 1 and b = 3
(ii) Let
1
2
3
OA

| |
|
=
|
|
\ .
and
4
0
1
OB

| |
|
=
|
|

\ .
.
3
2
4
AB

| |
|
=
|
|

\ .

Distance =
2 2 2
3 2 10
2 4 17
4 3 16
645
4.72
29 29
2 4 3
| | | | | |
| | |

| | |
| | |

\ . \ . \ .
= = =
+ +
(3 s.f.)
(iii) Vector equation of plane is
1 2 3
2 4 2
3 3 4
r s t
| | | | | |
| | |
= + +
| | |
| | |

\ . \ . \ .
, where , s t e
2 A sequence of positive real numbers
1
x ,
2
x ,
3
x , ... satisfies the recurrence relation
1
2 5
n n
x x
+
= + for
1 n > .
(i) If this sequence converges to l, determine the exact value of l. [2]
(ii) By using a sketch of the graph of 2 5 y x x = + and considering
1 n n
x x
+
, prove that

1 n n
x x
+
> if 0
n
x l < < ,

1 n n
x x
+
< if
n
x l > . [3]
(iii) Describe the sequence when
1
x l = . [1]

Solution:
(i) 2 5 l l = +

2
2 5 0 l l =

( )
2
2 2 4 5
2 24 2 2 6
1 6
2 2 2
l


= = = =
Since this is a sequence of positive real numbers,
0 l > .
Hence, 1 6 l = + .
(ii)

1
2 5
n n n n
x x x x
+
= +
From the sketch, if 0
n
x l < < ,
2 5 0
n n
x x + >

1
0
n n
x x
+
>

1 n n
x x
+
> (proved)

If
n
x l > ,
2 5 0
n n
x x + <

1
0
n n
x x
+
<

1 n n
x x
+
< (proved)

(iii) If
1
x l = ,

2 1
2 5 2 5 x x l l = + = + =
Similarly,
3
x l = and so on.
Thus,
n
x l = for all n
+
e .

0
x
y


3 (i) Sketch the graph of ln y x x = . By adding a suitable graph to the sketch, find the set of values
of x that satisfies
3 2 2
ln 5 1 0 x x x x + s . [4]

(ii) Using an algebraic method, solve the inequality
( )
2
18 6
1
1
x
x
x
+
s

. [3]
Solution:

(i)
3 2 2
ln 5 1 0 x x x x + s
3 2 2
ln 5 1 x x x x s
3 2 2
ln 5 1 x x x x s

2
1
ln 5 x x
x
s
Using G.C.,
x o | s s
0.512 6.91 x s s (3s.f.)

(ii)
( )
2
18 6
1
1
x
x
x
+
s



( )
( )( )
( )
2 2
6 1
18
0
1 1
x x
x x
+
s


( )
2
2
5 24
0
1
x x
x
+
s


2
5 24 0 and 1 x x x + > =
( )( ) 8 3 0 and 1 x x x + > =
8 or 3 x x s >




4 Given that
1
3 z i = + and
2
1 3 z i = , find
1 2
z z and ( )
1 2
arg z z . [2]

The complex number z satisfies the equation
1 2
z z z = . Sketch, on an Argand diagram, the locus
of point P representing the complex number z. [2]

Hence, find the least possible value of ( )
2
arg z z . [3]

5 A certain supermarket sells only three brands of health drinks namely Mud, Soil and Rock which are
priced at $2.00, $1.80 and $1.50 per bottle respectively.

Discounts are offered for each brand as follows:

20% discount when more than 80 bottles of Mud are bought;
15% discount when more than 50 bottles of Soil are bought;
10% discount when more than 30 bottles of Rock are bought.

Andy orders 200 bottles of health drinks from the supermarket for a company function. As the
preference for each brand differs among the colleagues in the company, he has to order at least 60
bottles of each of the 3 brands.

Andy spent $323.05 after an overall discount of $27.45.

Find the number of bottles of each brand Andy bought from the supermarket, explaining your answer
clearly. [7]

Solution:
Let , and M S Rbe the number of Mud, Soil and Rock Andy bought respectively.

We have the systems:

+ 200 (1) M S R + =
2 1.8 + 1.5 323.05 27.45 350.50 (2) M S R + = + =

Since Andy has to order at least 60 bottles of each brand, he must have bought less than 80 bottles of Mud.

2 0.85 1.8 + 0.9 1.5 323.05 (3) M S R + =

Solving (1), (2), (3) using GC gives 65; =60; 75 M S R = =

Andy bought 65 Mud, 60 Soil and 75 Rock from the supermarket.
6 The functions f and g are defined as follow:
2
f : x x , , 1 3 x x e s s ,
9
g:
x
x e

, , 0 x x e s .

Explain why gf exists. [2]

Express gf as a function in a similar manner and find its range in exact form. [4]

Determine whether gf has an inverse. [2]

Solution:
| | | |
1, 3 9, 0
f f
D R = = And
3
( , 0] ( , ]
g g
D R e

= =

Since | |
9, 0 ( , 0]
f g
R D = c = gf exist.

2
9
gf : , , 1 3
x
x e x x
+
e s s ,


From GC

18 9
,
gf
R e e

(
=



Since
10
y e

= cuts
2
9 x
y e
+
= at 2 points,
2
9
gf( )
x
x e
+
= 1 3 x s s is not 1-1 and hence has NO inverse.




7 Given that
1
1
+
=
r r
U
r
, show that n U
n
r
r
=

=1


(i) by using the method of differences, [3]

(ii) by mathematical induction. [4]
Hence, prove that n
r
n
r
<

=1 2
1
. [2]
Solution:
(i)
1 1 1
1
1 1 1
r
r r
U r r
r r r r r r

= = =
+ +
,
LHS = ( )

= =
=
+
n
r
n
r
r r
r r
1 1
1
1
1


= ( ) ( ) ( ) | | | | 1 2 1 ... 2 3 1 2 0 1 + + + + + n n n n

n = = RHS

(ii)
Let P
n
be the statement n U
n
r
r
=

=1
,
+
Z e n .
For n = 1, LHS =
1
1
1
1 0 1
r
r
U U
=
= = =

, RHS = 1 P
1
is true.
Assume that P
k
is true for some
+
Z e k , i.e.
1
k
r
r
U k
=
=

.
Need to show
1
1
1
k
r
r
U k
+
=
= +

.
LHS =
( ) 1
1
1 1 1
k
r k
r
U U k k k
+
=
+ = + + +


1 1 + = + + = k k k k = RHS.
P
k
is true P
k+1
is true.
Since P
1
is true and that P
k
is true P
k+1
is true, P
n
is true for all
+
Z e n .

n
r r r r r
n
r
n
r
n
r
=
+
<
+
=

= = = 1 1 1
1
1 1
2
1
.

8 Given that ( ) ( )
sin ln 1 2 y x = + , prove that ( ) ( )
2
2
2
d d
1 2 2 1 2 4 0.
d d
y y
x x y
x x
+ + + + = Find the
Maclaurins series for ( ) ( )
sin ln 1 2x + up to and including the term in x
3
. [7]

Hence, show that ( ) ( )
2
cos ln 1 2 1 2 x x + ~ . [2]
Solution:

2
cos(ln(1 2 )
1 2
dy
x
dx x
= +
+

(1 + 2x)
dy
dx
= 2cos(ln(1 + 2x))
(1 + 2x)
2
2
d y
dx
+ 2
dy
dx
= 2sin(ln(1 + 2x))
2
1 2x +

(1 + 2x)
2
2
2
d y
dx
+ 2(1 + 2x)
dy
dx
+ 4y = 0
(1 + 2x)
2
3
3
d y
dx
+ 4(1 + 2x)
2
2
d y
dx
+ 2(1 + 2x)
2
2
d y
dx
+ 8
dy
dx
= 0
or (1 + 2x)
2
3
3
d y
dx
+ 6(1 + 2x)
2
2
d y
dx
+ 8
dy
dx
= 0
x = 0, y = 0 ,
dy
dx
= 2 ,
2
2
d y
dx
= 4 ,
3
3
d y
dx
= 8

y = 2x 2x
2
+
3
4
3
x

2
2 4 4
dy
x x
dx
= +
Thus cos(ln(1 + 2x)) ~ (1 + 2x)(1 2x + 2x
2
) ~ 1 4x
2


9 By using the substitution 2x = sinu, show that


( )
2
1 2
2 0
1
d sin 2 2 1 4
16
1 4
k
x
x k k k
x

=

}
, where k
+
e . [5]

Hence, find the value of k, correct to 3 significant figures, such that


0.3
1 1
0 0
sin 2 d sin 2 d
k
x x x x x x

=
} }
. [4]

Solution:
2
2
1 4
x
dx
x
}

=
2
2
sin 1
cos
2
4 1 sin
d
u
u u
u
}

=
2
1
sin
8
d u u
}

=
1
(1 cos 2 )
16
d u u
}
=
1 sin 2
16 32
C
u
u +
=
1 2sin cos
16 32
C
u u
u +
=
( )
1 2
1
sin 2 2 1 4
16
x x x C

+
( )
2
1 2
0
2
1
sin 2 2 1 4
16
1 4
k
x
dx k k k
x

=
}





0.3
1 1
0 0
sin 2 sin 2
k
x x dx x x dx

=
} }

0.3
1
0
sin 2 x x dx

}
= 0.064706 (using a GC)

1
0
sin 2
k
x x dx

=
}
2 2
1
2
0
0
2
sin 2
2
2 1 4
k
k
x x
x dx
x

}
(


=
2
1 1 2
1
sin 2 (sin 2 2 1 4 )
2 16
k
k k k k


=
2 2
1
1 1 4
( )sin 2
2 16 8
k k k
k


+
Thus
2 2
1
1 1 4
( )sin 2
2 16 8
k k k
k


+ = 0.064706

Using a GC, k = 0.44351 ~ 0.444 (correct to 3 sf)


10. (a) Find all the asymptotes of
( )( )
2
2
1
x x
y
x x
+
=
+
, where 1 < .
Hence, sketch the curve. (Coordinates of the turning points are not required.) [4]

(b)









The diagram shows a sketch of the graph of ( ) f y x = . Sketch, on separate clearly labeled
diagrams, the graphs of
(i)
( )
1
f
y
x
= , [3]
(ii) ( ) f ' y x = . [3]
Solution:
(i)
( )( )
( )
( )( )
2
3
2
1
1 1
x
x x
y
x x x x


+
+
= = +
+ +

Asymptotes: = x , 1 = x , 1 = y















y
x
O
5
1
1


x = 1

y
x
y = 1

-2 O
b(i)














(ii)











y
x
O
5
1
1




y
x
O 2
1
11 (a) The terms u
1
, u
2
, u
3
, form an arithmetic sequence with first term a and having non-zero
common difference d.

(i) Given that the sum of the first 8 terms of the sequence is 98 more than u
29
, find the first
term of the sequence. [2]

(ii) If u
15
is the first term in the sequence greater than 196, show that 13 14 d < s . [3]

(b) The terms v
1
, v
2
, v
3
, form a geometric sequence with common ratio r. Another sequence {w
n
}
is then defined by w
n
= v
2n 1
+ v
2n
for all positive integers n.

(i) Show that w
n
is a geometric sequence with common ratio r
2
. [2]

(ii) Given that v
1
= 4 and the sum of all the odd-numbered terms w
1
, w
3
, w
5
, in the sequence
{w
n
} is
32
15
, find the value of r and
1
r
r
v

. [4]
Solution:
(a) (i) S
8
= u
29
+98
4(2a + 7d) = a + 28d + 98
7a = 98
a = 14
(ii) Given that u
14
s 196 and u
15
> 196,
14 + 13d s 196 and 14 + 14d > 196
Hence d s 14 and d > 13
i.e. 13 < d s 14.


(b) (i)
w
n
w
n 1
=
v
2n 1
+ v
2n
v
2n 3
+ v
2n 2
=
ar
2n 2
+ ar
2n 1
ar
2n 4
+ ar
2n 3
= r
2

Hence the sequence {w
n
} is a g.p. with common ratio r
2
.
(ii) w
1
+ w
3
+ w
5
+ =
4 + 4r
1 r
4 =
32
15

From GC, r =
1
2
(since r = 1)
Hence
r = 1

v
n
=
4
1
\

|
.
|
|

1
2
=
8
3
.
12. (a) A curve C is given parametrically by the equations,


1
=
t
t
x ,
1
) 1 (
3
2 2

=
t
t t
y , where 1 t = .

Find the exact area bounded by the curve C, the x axis and the line
1
3
x = . [5]


(b) The graph y = e
x
, for 1 0 s s x is shown in the diagram below. The region bounded by the curve,
the axes and the line x = 1, is rotated through four right angles about the x-axis to form a solid.















(i) The shaded region A forms a circular disc when it is rotated through four right angles about
the x-axis. Show that the volume of this disc is
4
n
e
n
t
unit
3
. [2]
(ii) Circular discs, each of width
n
1
, are used to approximate the volume of the solid formed
above. By considering the volume of the discs, show that for all n
+
e ,
4
2
1
1
n
e
n
e
t
| |

|
|
\ .
<
( )
2
1
2
e
t
[4]
x
y


y = e
x
A


Solution:
(a) When x =
3
1
, t = 0.5, and y = 0.5.
When x = 0, y = 0, and t = 0.

Area bounded by the curve = |
}
3
1
0
dx y | = | dt
dt
dx
y
}

2
1
0
|
= |
( )
dt
t t
t t
}

|
|
.
|

\
|

2
1
0
2 3
2 2
1
1
1
) 1 (
|
= | dt
t
t
}

2
1
0
3
2
1
|
= | dt
t
t
}


0
2
1
3
2
1
3
3
1
|
= | | |
0
2
1
3
| 1 | ln
3
1

t |
=
8
9
ln
3
1
.

(b)(i) Required volume
( )
2 4
2
2
1
n n
r h e e
n n
t
t t
| |
= = =
|
\ .


(ii) Volume of all the circular discs
=
( )
2 2 2
1 2 1
2
0
1 1 1 1
n
n n n
e e e e
n n n n
t

(
| | | | | |
( + + + +
| | |
(
\ . \ . \ .

=
0 1 2 1
2 2 2 2
n
n n n n
e e e e
n
t

(
| | | | | | | |
( + + + +
| | | |
(
\ . \ . \ . \ .


=
2
2
1
1
n
n
n
e
n
e
t
(
| |
(
|
(
\ .
(
(

(


=
2
2
1
1
n
e
n
e
t
| |

|
|
\ .


Volume of solid generated =
( )
1
1
2 2 2
0
0
d 1
2 2
x x
e x e e
t t
t ( = =
}
.
As each disc is smaller than the volume of the part of the solid that it is supposed to approximate, the
volume of all the circular discs is always smaller than the volume of the solid formed.

Hence,
2
2
1
1
n
e
n
e
t
| |

|
|
\ .
<
( )
2
1
2
e
t
for all n
+
e .


Temasek Junior College H2 9740 Paper 2 Solutions


Section A: Pure Mathematics [40 marks]


1
(i)
dx
dt
= 2t ,
dy
dt
=
1
(t 2)
2
dy
dx
=
1
2t(t 2)
2

(ii) At P, 6 =
1
t 2
+ 5 t = 3

Gradient of normal at P = 2(3)(3 2)
2
= 6
Equation of normal at P is y 6 = 6(x 4)
i.e. y = 6x 18.

(iii) When the normal at P cuts C again,

1
t 2
+ 5 = 6(t
2
5) 18
From GC, since t = 2, t = 2.97 or t = 1.97 (3sf)



2
(i)
1
p(1 0.005p)
=
A
p
+
B
(1 0.005p)


i.e. 1 = A(1 0.005p) + Bp
p = 0: A = 1; p = 200, B = 0.005
Thus
1
p(1 0.005p)
=
1
p
+
0.005
(1 0.005p)
=
1
p
+
1
(200 p)
.

(ii)
dp
dt
= ap (1 0.005p)

]
(
1
p(1 0.005p)
dp =
]
(a dt
]
(
1
p
+
1
(200 p)
=
]
(a dt

ln |p| ln |(200 p)| = at + C
ln

p
200 p
= at + C

p
200 p
= Be
a t

p =
200Be
a t
1 + Be
a t
When t = 0, p = 150 B = 3
When t = 5, p =
600e
1 + 3e
a =
1
5

Thus p =
600e
0.2t
1 + 3e
0.2t

(iii) As t , p 200. the limiting population is 200,000.



3 Complex Numbers



4
(a)(i) Angle between
\

|
.
|
|
1
2
1
and
\

|
.
|
|
1
0
2
= cos
1

\

|
.
|
|
\

|
.
|
|
1
2
1

\

|
.
|
|
1
0
2
1
2
+ 2
2
+ (1)
2
1
2
+ 0
2
+ (2)
2

= cos
1

3
30
= 56.8
Angle between l and H is 90 56.8 = 33.2 (1d.p.)

(ii) At the point of intersection P,
\

|
.
|
|
|
1 +
1
3 2

\

|
.
|
|
1
2
1
= 4
1 + + 2 3 + 2 = 4 =
4
3

The point of intersection is (
7
3
, 1,
1
3
).

(iii) P is the midpoint of line segment TJ.
OJ

= 2OP

OT

=
2
3

\

|
.
|
|
7
3
1

\

|
.
|
|
1
1
3
=
1
3

\

|
.
|
|
11
3
7


(b) Considering intersection of 3 planes:

x y + z = 2
4x 2y + z + 3
3x y + 3z = 4

From GC, x =
1
2
+
1
2
z, y =
5
2
+
3
2
z, z = z
Thus the intersection of the 3 planes is a line with equation r =
\

|
.
|
|
|

1
2

5
2
0
+
\

|
.
|
|
|
1
2
3
2
1
.
Hence, they intersect at infinitely many points.
When z = 3, = 3 and the position vector of the common point is i + 2j + 3k.


Section B: Statistics [60 marks]
5 (i) No. of ways =
4
C
2

5
C
3
= 60
(ii) No. of ways =
4
C
1
4! = 96
(iii) No. of ways =
4
P
4
+
5
P
4
= 144


6












Let x = probability an alumni member found a job he did not enjoy given that he did not enjoy
his course of study.

(i) P(an alumni member found a job which he enjoyed)
= (0.6 0.7) + 0.1 = 0.52
(ii) From tree diagram, 0.4x = 0.3 x = 0.75
(iii) P(an alumni member did not enjoy his course of study| he found a job which he enjoyed)
=
0.1
0.52
=
5
26




7 Let X = number of customers per day. Then X ~ Po()
Since P(X = 3) = P(X = 4), e

3
3!
= e

4
4!
.
Hence = 4.

Let Y = number of customers per week. Then Y ~ Po(28)
From GC, P(Y > 30) = 1 P(Y < 30) = 0.30965 (shown).

Let W = number of weeks during which there are more than 30 customers.
Then W ~ B(52, 0.30965)

Since n = 52 (large), np = 16.102 >5 and n(1 p) = 35.898 >5,
W ~ N(16.102, 11.116) approximately.

P(8 < W < 12) = P(7.5 < W < 12.5) (continuity correction)
= 0.135 (3sf)


Enjoyed
study
Did not
enjoy study
0.6
0.4
0.7
0.3
Enjoyed
job
Enjoyed
job
Did not
enjoy job
Did not
enjoy job
1 x
x
8 Let X = the height of a male citizen. Then X ~ N(170, o
2
) where o is the standard deviation.
Since P(X > 165) = 0.9,
Thus P
\
|
.
|
Z >
165 170
o
= 0.9
From GC,
5
o
= 1.28155 o = 3.90 (3sf)

Let Y = the height of a female citizen. Then Y ~ N(162, 9.65
2
).
Then X Y ~ N(8, 108.344364)
P(|X Y| > 6) = 1 P(6 < X Y < 6)
= 0.665 (3sf)

Let W = the number of couples out of 10 with heights differing by less than 6cm.
Then W ~ (10, 0.3345)
P(W < 4) = 0.784


9 (a) H
0
: = 28
H
1
: > 28
x = 29.4 , and s
2
=
15
14
(2.38
2
) = 6.069
Assuming that the burning time follows a Normal distribution,
since n is small and o is unknown,
Test statistic =
x
s/ n
~ t
14
.
If H
0
is true, = 28.
From GC, p-value = 0.0225
Hence the smallest integer o to reject H
0
at o% level of significance is 3.
(b) H
0
: = 28
H
1
: = 28
Since n = 100 is large and o is known,
Test statistic =
x
o/ n
~ N(0,1).
If H
0
is true, = 28.
For H
0
to be rejected at 2% level of significance, z
cal
=

x 28
3.2/10
> 2.3263
Thus x < 27.3 or x > 28.7.



10 (i) Stratified sampling was used. This method gives a good representative sample of the
population.

(ii) x =
227.4
60
= 3.79 , s
2
x
=
1
59\
|
.
|
997.78
(227.4)
2
60
= 2.304
y =
273.6
72
= 3.8 , s
2
y
=
209.35
71
= 2.95

(iii) Since n is large for both samples, by Central Limit Theorem,
X

~ N(3.79,
2.304
60
) and Y

~ N(3.8,
2.95
72
) approximately.
Hence X

~ N(0.01, 0.079352)
P(X

> 0) = 0.486


11











The least squares regression line of y on x is a line y = a + bx where a and b are chosen such that
the sum of squares of deviation in y-values (y
i
a bx
i
)
2
is minimised.

(i) From the given data, t = 50 and v =
396 + k
9
.
Since the regression line passes through ( t , v),
396 + k
9
= 19.19 + 0.625(50)
Thus k = 58 (nearest integer).
From GC, r = 0.922 (3sf)

(ii) From GC, sum of squares of residues = 410.472 (3d.p.) or 410 (3sf)

(iii) When t = 90, v = 19.19 + 0.625(90) = 75.44
This is not possible as there is only 70 ml of perfume in the bottle.

(iv) Based on the scatter plot of the data, the most appropriate model is (II)
From GC, the regression line of v on
1
t
is v = 69.82
616.44
t


(v) Based on the second model, when t = 90, v = 69.82
616.44
90
= 62.97
Compared to the result in (iii), the result obtained here is closer to the experimental
value of 67, and is also less than the maximum possible value of 70.




y
x
-
-
-
-
-
-
-
-
-
-
e
n

(x
n
, y
n
)
-
e
3

(x
3
, y
3
)
-
e
2

(x
2
, y
2
)
(x
1
, y
1
)
e
1

You might also like